The question bank may take some time to load… Just enough time to stretch, blink a few times, and question your life choices — but not too long, we promise!
We recommend going Full Screen for the best experience. Have Fun !
Report a question
Endo – Pharmacology
Compiled Questions from Modular + Annual Exams on Endo – Pharmacology
Which class of drugs has the strongest evidence for reducing LDL cholesterol and cardiovascular events, especially in patients with diabetes?
1 / 81
Tags:
2019
A 50-year-old male with Diabetes Mellitus comes to the clinic for a regular medical checkup. On examination, the patient has a BMI of 29, bilateral and yellow plaques on the eyelids. The physician orders a lipid panel to check for blood levels of LDL-C and triglycerides. If reports show an increase in LDL-C, which of the following drugs should the patient be started on?
Clinical Context:
The patient has xanthelasma , which is often associated with dyslipidemia , especially high LDL cholesterol.
Diabetes mellitus increases cardiovascular risk; thus, LDL lowering is crucial.
Increased LDL-C is the main lipid abnormality to target to reduce cardiovascular events.
Drug Options Analysis:
1. Ezetimibe
Works by blocking intestinal absorption of cholesterol .
Used as add-on therapy or if statins are contraindicated/intolerant.
Not first-line monotherapy for LDL reduction.
2. Bezafibrate
A fibrate used primarily to reduce triglycerides and increase HDL.
Less effective for LDL reduction.
Not first-line for high LDL-C.
3. Atorvastatin
✅ Correct choice
A statin , which inhibits HMG-CoA reductase and decreases cholesterol synthesis.
First-line therapy for lowering LDL-C.
Proven to reduce cardiovascular morbidity and mortality, especially important in diabetics.
4. Alirocumab
A PCSK9 inhibitor , monoclonal antibody that dramatically lowers LDL-C.
Reserved for patients with statin intolerance or those who need additional LDL lowering beyond statins.
Not first-line due to cost and administration (injection).
5. Fenofibrate
Another fibrate drug, mainly lowering triglycerides.
Like bezafibrate, less effect on LDL-C.
Not first-line for high LDL.
Summary:
Statins (Atorvastatin) are the first-line drugs to treat elevated LDL cholesterol, especially in high-risk patients like those with diabetes.
Final Answer:
✅ Atorvastatin
Think carefully about which condition results from the absence of a natural hormone and would therefore benefit from its replacement. Which one among the options reflects a problem of water imbalance rather than sugar, airways, or heart rate ?
2 / 81
Tags:
2016
In which of the following conditions is vasopressin indicated?
Vasopressin, also known as antidiuretic hormone (ADH) , is a hormone synthesized in the hypothalamus and secreted by the posterior pituitary gland. It plays a crucial role in water reabsorption in the kidneys and vascular tone regulation .
There are two major types of vasopressin receptors :
V1 receptors: found in vascular smooth muscle — cause vasoconstriction .
V2 receptors: found in the kidneys — stimulate insertion of aquaporin-2 channels in the collecting ducts, leading to increased water reabsorption and concentrated urine .
✅ Why the Correct Option is Right:
Central diabetes insipidus is caused by a deficiency of vasopressin (either due to damage to the hypothalamus or the posterior pituitary). The hallmark symptom is polyuria (excessive urination) with dilute urine , leading to polydipsia (excessive thirst) and a risk of dehydration and hypernatremia .
Because central DI results from low or absent ADH , vasopressin replacement (or its synthetic analog desmopressin) is the treatment of choice. Administering vasopressin compensates for the hormone deficiency, reducing urine output and restoring fluid balance.
❌ Why the Other Options Are Wrong:
Bronchospasm Bronchospasm is typically treated with bronchodilators like β2-agonists (e.g., salbutamol). Vasopressin has no role in bronchodilation and may worsen conditions by causing vasoconstriction that could reduce perfusion.
Diabetes mellitus This is a glucose metabolism disorder , not related to ADH. The confusion may stem from the similar names (“diabetes insipidus” vs. “diabetes mellitus”), but their causes and treatments are entirely different . Vasopressin has no role in managing blood glucose.
Tachycardia Although vasopressin can increase blood pressure by vasoconstriction (via V1 receptors), it does not directly treat tachycardia . In fact, by increasing afterload and blood pressure, it could potentially worsen certain types of tachycardia. Management of tachycardia depends on its cause and may involve beta-blockers, vagal maneuvers, or antiarrhythmics , not vasopressin.
Hypertension Vasopressin causes vasoconstriction , which can raise blood pressure , so it is contraindicated in hypertension . Administering vasopressin here could worsen the condition , not improve it.
Which drug on this list is known to contain a high level of iodine and directly interferes with thyroid hormone synthesis and metabolism — often used in cardiac care?
3 / 81
Tags:
2019
Which of the following drugs can cause drug-induced hypothyroidism?
Hypothyroidism can occur as a side effect of several medications, but Amiodarone is a particularly important drug to recognize for its thyroid-related toxicity .
📌 Why Amiodarone Causes Hypothyroidism:
Amiodarone is a class III antiarrhythmic drug used to treat ventricular and supraventricular arrhythmias.
It affects the thyroid through multiple mechanisms:
High Iodine Content :
Amiodarone contains about 37% iodine by weight , and each 200 mg tablet delivers ~75 mg of iodine , which is hundreds of times the daily recommended intake.
This iodine load can cause Wolff-Chaikoff effect — an acute inhibition of thyroid hormone synthesis due to excess iodine.
Inhibition of Peripheral Conversion :
Direct Cytotoxicity :
💡Note: Interestingly, Amiodarone can also cause hyperthyroidism , particularly in iodine-deficient areas (called Amiodarone-induced thyrotoxicosis ), but hypothyroidism is more common in iodine-sufficient regions.
❌ Explanation of Incorrect Options:
Nivolumab 🔴 A checkpoint inhibitor (PD-1 antagonist) used in cancer immunotherapy. It can cause thyroid dysfunction , usually transient thyroiditis leading to hyperthyroidism , then hypothyroidism — but this is less common and immune-mediated, not a direct pharmacologic effect like Amiodarone.
Alemtuzumab 🔴 A monoclonal antibody used in MS and certain leukemias. It may cause autoimmune thyroid disease , but hypothyroidism is not its most consistent effect, and not through direct thyroid toxicity.
Aspirin 🔴 Does not cause hypothyroidism . In fact, high doses can displace thyroid hormone from binding proteins , temporarily increasing free T4.
Pembrolizumab 🔴 Like Nivolumab, it’s a PD-1 inhibitor. It may cause immune-related thyroiditis , but again, this is autoimmune and less predictable .
To treat a condition caused by excess of a hormone that’s normally kept in check by dopamine, what class of drug would you use — one that blocks dopamine, or one that mimics it?
4 / 81
Tags:
2018
First-line therapy for prolactinoma is:
Octreotide
Metoclopramide
Acetazolamide
Dorzolamide
Bromocriptine
What is a Prolactinoma ?
🎯 Treatment Goals:
Normalize prolactin levels
Shrink the tumor
Restore fertility/menstrual function or sexual function
💊 First-Line Therapy: Dopamine Agonists
Dopamine inhibits prolactin secretion.
Why it’s correct:
Bromocriptine is a dopamine D2 receptor agonist
Used as first-line therapy for prolactinomas, especially in pregnant or childbearing women
Can be taken orally
Cabergoline (longer-acting, fewer side effects) is now often preferred, but bromocriptine remains a classic first-line choice
❌ Why the Other Options Are Incorrect:
Octreotide
Metoclopramide
A dopamine antagonist , actually increases prolactin levels — can cause hyperprolactinemia as a side effect
Contraindicated in prolactinoma
Acetazolamide
A carbonic anhydrase inhibitor , used for glaucoma , metabolic alkalosis , altitude sickness — no role in prolactinoma
Dorzolamide
Consider which anti-thyroid drug can lead to a serious side effect involving the bone marrow , resulting in low white blood cell count . This condition can present suddenly with sore throat and fever , making it a medical emergency .
5 / 81
Tags:
2017
Which of the following anti-thyroid drug is responsible for causing sore throat with fever? Total leukocyte count is also low.
Methimazole is a thionamide anti-thyroid drug used in hyperthyroidism treatment.
Its mechanism : Inhibits thyroid peroxidase , preventing the organification of iodine and thyroid hormone synthesis .
Serious Side Effect:
Agranulocytosis is the most feared side effect of methimazole (can also occur with propylthiouracil but is more common with methimazole).
It involves severe reduction in neutrophils (granulocytes) → very low WBC count .
Presents as:
Management:
Why the Other Options Are Incorrect:
Option
Why It’s Incorrect
Propylthiouracil
❌ Can also cause agranulocytosis, but methimazole is the more common culprit . Propylthiouracil is also associated with liver toxicity .
Levothyroxine
❌ Thyroid hormone replacement , not an anti-thyroid drug. It does not cause agranulocytosis.
Radioiodine
❌ Can lead to hypothyroidism or thyroiditis , but does not cause agranulocytosis .
Liothyronine
❌ A synthetic T3 preparation used for hypothyroidism or myxedema coma—not related to WBC suppression.
Summary:
Methimazole can cause agranulocytosis , presenting as sore throat, fever, and a low WBC count —a medical emergency requiring immediate attention.
If the body can’t hold onto water because of a missing hormone , what kind of replacement would restore that function?
6 / 81
Tags:
2020
In which of the following conditions is vasopressin indicated?
Vasopressin is another name for antidiuretic hormone (ADH) .
Its main job is to:
🧪 In Central Diabetes Insipidus , the problem is:
The pituitary gland doesn’t make enough ADH (vasopressin)
So, the body loses a lot of water through urine
This causes frequent urination and dehydration
👉 In this case, giving vasopressin (or desmopressin) replaces the missing hormone and helps the kidneys conserve water .
❌ Why the Other Options Are Wrong (Simple):
Hypertension
Diabetes mellitus
This is high blood sugar , not related to vasopressin or water balance.
The treatment involves insulin or oral sugar-lowering meds , not vasopressin.
Tachycardia
Bronchospasm
This patient’s bone density is not yet in the osteoporosis range, but her medication history puts her at high risk for fractures — which treatment would directly prevent bone resorption ?
7 / 81
Tags:
2023
A 65-year-old woman came to the clinic. She has been on oral steroids for chronic obstructive pulmonary disease (COPD) and had a DEXA scan showing a T score of -1.8. Which of the following would be the best management?
This patient has:
A T-score of -1.8 → osteopenia , not osteoporosis (osteoporosis = T ≤ -2.5)
A history of oral steroid use for COPD , which significantly increases her risk for fragility fractures
Glucocorticoid-induced bone loss is a serious concern even before T-score reaches -2.5 , especially if:
Alendronate , a bisphosphonate , is the first-line treatment in this setting. It:
Inhibits osteoclast-mediated bone resorption
Helps preserve bone density
Reduces vertebral and hip fracture risk
❌ Why the Other Options Are Less Appropriate Alone
Calcium and vitamin D supplements Essential for baseline bone health , but not sufficient alone to prevent fracture in glucocorticoid-induced osteopenia.
Abaloparatide A PTH analog , used for severe osteoporosis . Not first-line in osteopenia with moderate risk .
Raloxifene A SERM used primarily in postmenopausal women with osteoporosis and low breast cancer risk — less effective in steroid-induced cases.
All of these Overly broad; Alendronate is the best single answer per current guidelines.
Think about how thyroid hormones are synthesized—what steps occur before T3 and T4 are formed? Where would a drug intervene to reduce the production of active thyroid hormones without blocking their release?
8 / 81
Tags:
2019
Which of the following is the effect of propylthiouracil on thyroid hormone?
Propylthiouracil (PTU) is an antithyroid drug that inhibits thyroid hormone synthesis through two main mechanisms:
Inhibits thyroid peroxidase enzyme , which is responsible for:
Oxidation of iodide (I⁻ to I₂)
Iodination (organification) of tyrosine residues on thyroglobulin
Coupling of iodotyrosine molecules (MIT + DIT = T3, DIT + DIT = T4)
Also inhibits peripheral conversion of T4 to T3 (This action distinguishes PTU from methimazole.)
By blocking the oxidation and coupling steps, PTU directly decreases the synthesis of new T3 and T4 hormones in the thyroid gland.
Incorrect Answer Explanations:
Hypertrophy of thyroid follicles
Incorrect
This is a consequence of TSH stimulation , not a direct effect of PTU.
Chronic PTU use may lead to goiter formation due to feedback increase in TSH , but PTU itself does not cause hypertrophy directly .
Inhibit T3 release from thyroglobulin
Hyper activates Na⁺-I⁻ symporter
Inactivates thyroglobulin
Think about what “basal glucose control” implies:
Constant : It needs to work steadily, without big ups and downs.
24-hour : It needs to last a full day or close to it.
No pronounced peak : This is key, as a peak would mean a higher risk of hypoglycemia at that point, which is undesirable for background control.
9 / 81
Tags:
2024
A patient with diabetes mellitus is prescribed an insulin preparation designed to provide consistent 24-hour basal glucose control with no pronounced peak . Which of the following insulin types has this pharmacokinetic profile?
Insulin preparations vary by onset , peak , and duration . Here’s a quick comparison:
Insulin Type
Onset
Peak
Duration
Category
Lispro
~15 min
0.5–1.5 hrs
3–4 hrs
Rapid-acting ❌
Aspart
~15 min
1–3 hrs
3–5 hrs
Rapid-acting ❌
Glulisine
~15–30 min
1–2 hrs
3–5 hrs
Rapid-acting ❌
NPH
~1–2 hrs
4–10 hrs
12–18 hrs
Intermediate ❌
Glargine
~1–2 hrs
No real peak
~24 hrs
Long-acting ✅
➡️ Glargine is a long-acting insulin analog designed to mimic basal insulin secretion , maintaining steady glucose levels over a full day without a peak.
❌ Why the Other Options Are Incorrect:
Glulisine ❌ Rapid-acting; ideal for meals, not for 24-hour coverage.
Lispro ❌ Also rapid-acting; short duration of action.
NPH insulin ❌ Intermediate-acting; doesn’t last full 24 hours, and has a significant peak (risk of hypoglycemia).
Aspart ❌ Rapid-acting; used just before meals, not for basal control.
Think about why certain drugs are preferred for long-term management: Would a medication that needs conversion and has a long duration be more useful for stability — or a fast-acting one with a short duration?
10 / 81
Tags:
2016
Which of the following is inappropriate about levothyroxine?
🔹 What is Levothyroxine?
Levothyroxine is a synthetic form of T₄ (thyroxine) , the major hormone secreted by the thyroid gland. It is the mainstay of therapy for hypothyroidism and is converted to T₃ (triiodothyronine) in peripheral tissues, which is the active form.
🔬 Pharmacokinetics:
Half-life of levothyroxine (T₄): Approximately 7 days in euthyroid individuals (can be longer in hypothyroidism and shorter in hyperthyroidism)
Half-life of liothyronine (T₃): About 1 day — much shorter
This is why levothyroxine provides stable, long-term thyroid hormone replacement , whereas liothyronine acts faster but is not used routinely due to risk of rapid hormonal swings.
✅ Correct Statements (All True):
“It has longer half life than liothyronine”: True — levothyroxine (T₄) has a much longer half-life than liothyronine (T₃).
“Lower initial doses are administered in older patients with longstanding myxedema”: True — because starting high doses in such patients can trigger cardiac events (like angina or arrhythmias). Hence, a “start low, go slow” approach is used.
“It is usually the first form of choice in patients with hypothyroidism”: Absolutely true — levothyroxine is standard first-line therapy .
“It is slower acting than liothyronine”: Also true — levothyroxine needs conversion to T₃ and has a slower onset.
❌ Why “It has a short half-life” is inappropriate (False):
Levothyroxine’s long half-life is one of its greatest advantages — it allows for once-daily dosing and stable blood levels .
So stating that it has a short half-life is incorrect and does not reflect its clinical utility .
When asked about enzyme inhibition by glucocorticoids, think:What’s the earliest enzyme in the inflammation pathway? Block that, and you block everything downstream.
11 / 81
Tags:
2016
Which of the following enzymes is inhibited by glucocorticoids?
🔹 Mechanism of Glucocorticoids:
Glucocorticoids like hydrocortisone and prednisone are powerful anti-inflammatory and immunosuppressive agents . Their anti-inflammatory effect is largely due to their ability to suppress the entire arachidonic acid pathway at an early stage.
They do this by:
Inducing lipocortin (annexin-1) , which inhibits phospholipase A2
This prevents the release of arachidonic acid from membrane phospholipids
Without arachidonic acid, both prostaglandins and leukotrienes (which cause inflammation, pain, and swelling) are not produced
✅ Why “Phospholipase A2” Is Correct:
It’s the first step in the arachidonic acid cascade.
Inhibition of this enzyme blocks the production of both prostaglandins and leukotrienes , making glucocorticoids broad-spectrum anti-inflammatory agents.
This is more upstream than NSAIDs, which block cyclooxygenase only.
❌ Why the Other Options Are Incorrect:
Xanthine oxidase: Involved in uric acid synthesis , inhibited by allopurinol , not glucocorticoids.
Cyclooxygenase (COX): Blocked by NSAIDs (like ibuprofen, aspirin), not directly by glucocorticoids .
Histidine decarboxylase: Converts histidine to histamine , not a target of glucocorticoids.
5-lipoxygenase: Converts arachidonic acid into leukotrienes . It’s downstream of phospholipase A2, and while its activity is reduced indirectly (due to lack of arachidonic acid), glucocorticoids do not directly inhibit this enzyme.
Consider where in the body carbohydrates are broken down before they can be absorbed. Think about enzymes involved in the final steps of carbohydrate digestion. How might blocking these enzymes help control post-meal blood sugar spikes?
12 / 81
Tags:
2017
Which drug inhibits glucose absorption in the small intestine?
Glucosidase inhibitors are a class of oral antidiabetic drugs that specifically target the small intestine to inhibit carbohydrate absorption .
How do they work?
Alpha-glucosidase enzymes , located in the brush border of the small intestine , are responsible for breaking down complex carbohydrates (starches, disaccharides) into simple sugars (glucose) for absorption.
Glucosidase inhibitors (e.g., Acarbose , Miglitol ) block these enzymes , delaying carbohydrate digestion.
As a result, glucose absorption is slowed down , leading to a blunted postprandial (after-meal) rise in blood sugar levels .
This mechanism is especially helpful in managing postprandial hyperglycemia in type 2 diabetes mellitus .
Clinical effect:
Decreases post-meal blood glucose peaks
Minimal risk of hypoglycemia when used alone
Common side effects: flatulence, bloating, and diarrhea (due to undigested carbs fermenting in the colon)
Why the Other Options Are Incorrect:
Metformin
Metformin is a biguanide that works by:
It does not inhibit intestinal glucose absorption directly , although it may mildly reduce glucose uptake from the gut in some cases—but this is not its primary mechanism .
Insulin aspart
Rosiglitazone
Rosiglitazone is a thiazolidinedione (TZD) .
It works by activating PPAR-γ receptors , which increase insulin sensitivity in adipose tissue and muscle.
It has no effect on glucose absorption in the intestine .
Exogenous glucose
Think about which medication not only reduces thyroid hormone production but also carries a known risk of serious bone marrow suppression .
13 / 81
Tags:
2023
A patient who is taking antithyroid drugs for hyperthyroidism develops sore throat and fever. Her total leukocyte count is low. Which of the following drugs is she most likely taking?
Methimazole is an antithyroid drug used to treat hyperthyroidism by inhibiting thyroid hormone synthesis.
One of its serious adverse effects is agranulocytosis (a marked reduction in neutrophils).
Agranulocytosis presents with fever, sore throat, and low leukocyte count — exactly what this patient shows.
Patients on methimazole are advised to report sore throat and fever promptly.
Why the other options are incorrect:
Nadolol and Propranolol : Beta-blockers; used for symptomatic control in hyperthyroidism (reduce heart rate, tremors), but they do not cause agranulocytosis.
Radioactive iodine : Destroys thyroid tissue, but agranulocytosis is not a typical side effect.
Digoxin : Cardiac glycoside; used in heart failure and atrial fibrillation. Does not affect leukocyte count.
Consider how the body naturally regulates blood glucose after meals . There are incretin hormones (like GLP-1) that help stimulate insulin release and suppress glucagon , but they are rapidly broken down by a specific enzyme . Sitagliptin works by targeting this enzyme . What is that enzyme?
14 / 81
Tags:
2017
What is the mechanism of action of sitagliptin?
Mechanism of Action:
DPP-4 normally breaks down incretin hormones (GLP-1 and GIP).
Incretins increase insulin secretion (glucose-dependent) and decrease glucagon secretion after meals.
Sitagliptin inhibits DPP-4 , leading to:
Increased GLP-1 and GIP levels
Prolonged insulin secretion
Decreased glucagon release
Better postprandial glucose control
Why the Other Options Are Incorrect:
Option
Why It’s Incorrect
PPARγ agonists
❌ These are thiazolidinediones (e.g., pioglitazone), which increase insulin sensitivity , not inhibit DPP-4.
SGLT2 inhibitor
❌ These drugs (e.g., empagliflozin ) block renal glucose reabsorption , causing glucose loss in urine , not affecting incretins.
Alpha-glucosidase inhibitor
❌ These drugs (e.g., acarbose ) delay carbohydrate absorption in the intestine , unrelated to incretin pathways.
GLP-1 receptor agonist
❌ Drugs like exenatide and liraglutide mimic GLP-1 , but sitagliptin prolongs natural GLP-1 action by inhibiting DPP-4 .
Summary:
Sitagliptin is a DPP-4 inhibitor that increases incretin levels , leading to improved insulin secretion and reduced glucagon , helping control blood sugar in type 2 diabetes.
Consider which drug group is known for causing widespread metabolic changes , including increased appetite , altered fat distribution , and fluid retention — often prescribed for inflammation or autoimmune conditions.
15 / 81
Tags:
2019
While prescribing which one of the following drugs should physicians be aware that it might cause weight gain?
Steroids , particularly glucocorticoids like prednisone , are well known to cause weight gain as a side effect. The mechanisms include:
Increased appetite : Glucocorticoids stimulate appetite and lead to higher caloric intake.
Fluid retention : They cause sodium retention and water retention, contributing to increased body weight.
Fat redistribution : Chronic use leads to central obesity , moon face , and buffalo hump (Cushingoid features).
Insulin resistance : Promotes fat storage and hyperglycemia , further compounding weight gain.
This effect is dose-dependent and more commonly seen with long-term use .
❌ Incorrect Answer Breakdown:
Topiramate : Actually associated with weight loss , not gain. It is used off-label for obesity and in migraine prevention and epilepsy.
Fluoxetine : An SSRI antidepressant; often causes initial weight loss . While long-term use may lead to mild weight changes, it is not typically associated with significant weight gain.
Aspirin : A non-steroidal anti-inflammatory drug (NSAID) . It has no significant effect on weight .
Antibiotics : While some animal studies suggest antibiotics can alter gut flora and influence metabolism, in clinical human use , antibiotics are not directly associated with weight gain .
If you give too much of a hormone that increases metabolism, which system in the body is likely to show early signs of being over-revved?
16 / 81
Tags:
2018
What are the side effects of levothyroxine?
Levothyroxine is synthetic T4 (thyroxine), used to treat hypothyroidism by replacing deficient thyroid hormone.
However, if:
⚠️ Common Side Effects of Levothyroxine (Signs of Overdose):
System
Effect
Cardiovascular
Tachycardia , palpitations, arrhythmia, angina , AFib , even MI in elderly with heart disease
CNS
Anxiety, irritability, insomnia
GI
Diarrhea, increased appetite
Musculoskeletal
Tremors, heat intolerance
Metabolic
Weight loss, sweating
So tachycardia is a very common early sign of over-supplementation.
❌ Why the Other Options Are Incorrect:
Nephropathy ❌ → Not a known effect of levothyroxine. The drug does not damage the kidneys .
Vision loss ❌ → Not directly linked. Thyroid eye disease may occur in Graves’ disease , but that’s autoimmune — not caused by levothyroxine .
Myocardial infarction (MI) ❌ → Rare, but possible in high doses, especially in elderly or heart patients. However, tachycardia occurs much earlier and more commonly , making it the best answer here.
Neuropathy ❌ → Not a recognized side effect of levothyroxine. Hypothyroidism can cause neuropathy, but treatment with levothyroxine does not cause it .
Which class of drugs helps reduce blood glucose by acting on the kidneys , not the pancreas or liver?
17 / 81
Tags:
2023
Empagliflozin belongs to which class of drugs?
Empagliflozin is a member of the SGLT2 inhibitor class of antidiabetic drugs.
It works by blocking the SGLT2 transporter in the proximal renal tubules , reducing glucose reabsorption .
This leads to increased urinary glucose excretion , thereby lowering blood glucose levels .
It also provides cardiovascular and renal benefits , making it especially useful in patients with diabetes, heart failure , or chronic kidney disease .
❌ Why the Other Options Are Incorrect
DPP-4 inhibitor (e.g., sitagliptin) — increases incretin levels by preventing their degradation.
GLP-1 agonist (e.g., liraglutide) — mimics incretin hormone, enhancing insulin secretion and reducing appetite.
Biguanides (e.g., metformin) — reduce hepatic gluconeogenesis and increase insulin sensitivity.
SGLT1 inhibitor Primarily affects glucose absorption in the gut , not the kidney; empagliflozin is SGLT2-selective .
What would your brain and adrenal glands do if you were constantly taking high doses of hormones that they normally try to produce?
18 / 81
Tags:
2022
Which of the following results will the reports show in a 35-year-old patient taking pharmacological doses of steroidal antiinflammatory drugs for asthma?
Pharmacologic doses of steroidal anti-inflammatory drugs (such as prednisone ) provide exogenous glucocorticoids , which mimic the action of cortisol . These high levels of glucocorticoids exert negative feedback on the hypothalamic-pituitary-adrenal (HPA) axis , leading to:
↓ Corticotropin-releasing hormone (CRH) from the hypothalamus
↓ Adrenocorticotropic hormone (ACTH) from the anterior pituitary
↓ Endogenous cortisol production from the adrenal cortex
Even though cortisol is high pharmacologically , the body detects high levels and shuts down its own CRH and ACTH , resulting in low endogenous cortisol levels .
If you stop steroids abruptly, the body cannot immediately produce cortisol, leading to adrenal insufficiency .
❌ Why the Other Options Are Incorrect:
High ACTH, low cortisol : Seen in primary adrenal insufficiency (e.g., Addison’s disease), not in steroid use.
High CRH, low cortisol : Seen in secondary adrenal insufficiency due to pituitary dysfunction—not due to exogenous steroids.
Low CRH, high cortisol : Would occur early during steroid use, but cortisol here refers to endogenous levels , which would be low over time.
High CRH, high cortisol : Seen in Cushing disease (ACTH-producing tumor) , not steroid use.
Among the options, which one would be useful if you needed a moderately quick effect without the extreme speed of a meal-time insulin — something you might use before a meal but not immediately before?
19 / 81
Tags:
2018
Which of the following insulin preparations is short-acting insulin?
Insulin is classified based on how quickly it starts working (onset ), when its effect is strongest (peak ), and how long it lasts (duration ).
Type
Onset
Peak
Duration
Examples
Rapid-acting
~15 mins
30–90 mins
3–5 hours
Lispro, Aspart, Glulisine
Short-acting
30–60 mins
2–4 hours
5–8 hours
Regular insulin
Intermediate-acting
1–2 hrs
4–12 hours
12–18 hours
NPH (Isophane insulin)
Long-acting
1–2 hrs
Minimal peak
Up to 24 hours
Detemir, Glargine, Degludec
🔬 Why “Regular insulin” is Correct:
It is the classic short-acting insulin .
Often used in IV form for emergencies like diabetic ketoacidosis (DKA) .
Onset and peak times are slower than rapid-acting insulins but faster than intermediate- or long-acting ones.
It’s also the only insulin type that can be given IV in acute settings.
❌ Why the Other Options Are Incorrect:
🔻 Detemir
🔻 Aspart
🔻 Lispro
🔻 Glulisine
If a patient presents with overstimulation of every parasympathetic system—salivating, sweating, urinating, and slowing their heart—what class of chemicals might be the culprit, and how can we regenerate the enzyme they’ve blocked?
20 / 81
Tags:
2020
A 12-year-old boy is found in a closed room with increased salivation from the mouth and sweating profusely. His pulse is 50 beats/minute and blood pressure is 100/60 mm Hg. Which of the following is the most appropriate antidote?
📌 Clinical Picture Suggests: Organophosphate Poisoning
Common in pesticide exposure (e.g., in closed rooms, farms, stored chemicals)
Organophosphates inhibit acetylcholinesterase , leading to:
🧬 Cholinergic Symptoms: DUMBBELSS Mnemonic
Symptom
Mechanism
D – Diarrhea
Muscarinic
U – Urination
Muscarinic
M – Miosis
Muscarinic
B – Bradycardia
Muscarinic
B – Bronchorrhea
Muscarinic
E – Emesis
Muscarinic
L – Lacrimation
Muscarinic
S – Salivation
Muscarinic
S – Sweating
Muscarinic
→ This patient has salivation , sweating , and bradycardia , which all fit.
🧪 Treatment of Organophosphate Poisoning:
Atropine — blocks muscarinic effects (salivation, bradycardia)
✅ Pralidoxime (2-PAM) — reactivates acetylcholinesterase , reversing both muscarinic and nicotinic effects
Supportive care (e.g., oxygen, fluids)
❌ Why Other Options Are Incorrect:
Option
Why Incorrect
Vitamin K
Antidote for warfarin toxicity , not relevant here
Flumazenil
Antidote for benzodiazepine overdose (e.g., drowsiness, coma), not cholinergic toxidrome
Warfarin
An anticoagulant , not an antidote
Propranolol
A beta-blocker — would worsen bradycardia
Imagine a hidden hormonal condition that depends on life-saving daily medication. What if a new drug secretly speeds up the breakdown of that medication — and no one notices until it’s too late?
21 / 81
Tags:
2018
A 45-year-old patient has been prescribed the 4-drug anti-tuberculous regimen for the next 3 months following a diagnosis of pulmonary tuberculosis. However, a few days later it is reported that the patient expired. What underlying undiagnosed condition most likely resulted in the demise of this patient?
The 4-drug anti-TB regimen typically includes:
Isoniazid (INH)
Rifampin
Pyrazinamide
Ethambutol
Among these, Rifampin is the most relevant here — it’s a potent inducer of liver enzymes (CYP450) .
💣 So what’s the danger with Rifampin in Addison’s disease?
Addison’s disease is a form of primary adrenal insufficiency , where the adrenal glands don’t produce enough cortisol and aldosterone . These patients are often on steroid replacement therapy (like hydrocortisone or prednisone).
Here’s the key issue:
Rifampin induces liver enzymes , which increase the breakdown of steroids .
This causes steroid levels to drop dangerously , even if the patient is still taking their usual dose.
The patient decompensates quickly , leading to an Addisonian crisis — which is life-threatening and marked by:
Hypotension
Hypoglycemia
Hyponatremia
Hyperkalemia
Shock → Death
So if Addison’s disease is undiagnosed , and Rifampin lowers cortisol even more, the patient can suddenly crash and die .
❌ Why the Other Options Are Wrong:
Congenital adrenal hyperplasia ❌ → A childhood disorder; doesn’t usually present this way in a 45-year-old unless already diagnosed and managed.
Hyperaldosteronism ❌ → Causes hypertension , hypokalemia — not sudden collapse or vulnerability to Rifampin.
Hyperparathyroidism ❌ → Involves calcium imbalance , not adrenal steroids or crisis.
Hypoaldosteronism ❌ → Can cause hypotension and hyperkalemia, but not as rapidly fatal or directly affected by Rifampin as Addison’s (which involves both cortisol and aldosterone ).
When a diabetic patient is overweight and on a high-dose secretagogue with poor glucose control, which agent targets insulin resistance and also benefits weight and cardiovascular risk?
22 / 81
Tags:
2020
A 42-year-old woman comes to the clinic for evaluation of uncontrolled blood sugar. She has a history of type 2 diabetes mellitus and takes glimepiride 20 mg every day before breakfast. Her BMI is 32 kg/m2 and HbA1c is 8.4%. Which of the following is the best treatment option for this patient?
📌 Step-by-Step Clinical Reasoning:
1. Target HbA1c in T2DM:
2. Current Medication:
Glimepiride is a sulfonylurea that increases insulin secretion.
She’s already on a very high dose (20 mg/day) — near the maximum recommended dose (8–10 mg/day) .
➤ So increasing glimepiride further is not safe or effective .
3. Next Step in Management:
Metformin is the first-line agent in almost all cases unless contraindicated (e.g., renal failure, liver disease).
Especially beneficial in:
Obese patients (like this one, BMI 32)
Has weight-neutral or weight-loss effects
Improves insulin sensitivity
Low cost and well-studied
❌ Why Other Options Are Incorrect:
Stop glimepiride and start insulin – ❌ Not necessary yet . Insulin is generally reserved for very high HbA1c (>10%), symptomatic hyperglycemia , or oral failure . This patient can still benefit from combination oral therapy.
Add DPP-4 inhibitor – ❌ Possible, but metformin is preferred first , both for efficacy and cost-effectiveness. DPP-4s are generally add-ons after metformin .
Increase the dose of glimepiride – ❌ Already at max dose ; further increase won’t help and increases risk of hypoglycemia .
Add on thiazolidinedione (e.g. pioglitazone) – ❌ Can improve insulin sensitivity, but weight gain is a concern in obese patients. Also linked to fluid retention , heart failure , and fracture risk .
Think of the most effective class of drugs that directly targets LDL cholesterol , especially in patients with existing cardiovascular disease .
23 / 81
Tags:
2018
A person with cardiovascular disease wants to decrease his low-density lipoprotein (LDL) cholesterol level. What would be the preferred suggestion?
Statins are the first-line treatment to lower low-density lipoprotein (LDL) cholesterol , especially in patients with cardiovascular disease (CVD) or high risk.
🔬 How Statins Work:
Inhibit HMG-CoA reductase , the rate-limiting enzyme in cholesterol synthesis.
Increase LDL receptor expression in the liver.
Lower LDL by 30–60%, depending on dose and type.
Also provide pleiotropic effects : improve endothelial function, stabilize plaques, and reduce inflammation.
❌ Why Other Options Are Less Preferred:
Niacin : Raises HDL and lowers triglycerides modestly. Not as effective or preferred for lowering LDL. Also has side effects (flushing, hepatotoxicity).
Fibrates : Primarily lower triglycerides and modestly raise HDL. Not effective at lowering LDL.
Exercise : Improves cardiovascular health and modestly helps with HDL and weight, but not sufficient alone to significantly reduce LDL in high-risk patients.
Weight loss : Helps improve lipid profile, but statins are still necessary for patients with established CVD to reach LDL targets.
💡 Clinical Tip:
In patients with established CVD, guidelines strongly recommend high-intensity statin therapy unless contraindicated.
If a medication contains a high concentration of iodine and is prescribed to cardiac patients, think about how excess iodine might disrupt thyroid hormone synthesis.
24 / 81
Tags:
2018
A 62-year-old male presents to the clinic with complaints of weight gain and lethargy. He suffers from hypertension and ischemic heart disease, for which he is taking drugs. His TSH and FT4 levels are 21 mIU/L and 0.02 ng/dL, respectively. Considering drug-induced hypothyroidism, which of the following drug might be the most common cause?
The patient’s lab results show markedly elevated TSH and very low free T4 (FT4) , which is consistent with primary hypothyroidism . The question asks about drug-induced causes , particularly in the context of hypertension and ischemic heart disease — common indications for amiodarone use.
📌 Why Amiodarone?
Amiodarone is a class III antiarrhythmic drug commonly used in patients with cardiac arrhythmias and ischemic heart disease .
It contains a high iodine content — approximately 37% by weight , and each 200 mg tablet delivers around 75 mg of iodine .
It can cause both hypothyroidism and hyperthyroidism .
🧪 Mechanism of Amiodarone-Induced Hypothyroidism (AIH):
Iodine overload → inhibits thyroid hormone synthesis (Wolff–Chaikoff effect)
Can directly inhibit 5’-deiodinase , reducing T4 to T3 conversion
AIH is more common in iodine-sufficient areas
❌ Why the Other Options Are Incorrect:
Interferon → Can induce autoimmune thyroiditis, but is used for hepatitis C or malignancies , not for heart disease.
Sulphasalazine → An anti-inflammatory used in ulcerative colitis and rheumatoid arthritis ; rarely causes hypothyroidism.
Interleukin blockers → Immunomodulators used in autoimmune diseases , not typically associated with hypothyroidism or used in heart disease.
Phenylbutazone → An NSAID with rare thyroid side effects, largely obsolete due to side effects; not indicated for cardiac conditions.
Among the hormones that influence digestion, which one specifically targets the pancreas to stimulate enzyme secretion in response to fats and proteins? Think about who tells the pancreas to “get to work.”
25 / 81
Tags:
2018
Somatostatin inhibits which of the following hormones to decrease the pancreatic exocrine secretions?
Somatostatin is a potent inhibitory hormone secreted by D-cells in the stomach, intestine, and delta cells of the pancreas. It broadly suppresses endocrine and exocrine secretions throughout the gastrointestinal tract.
🔬 What does somatostatin do?
💡 Why CCK is the correct answer:
CCK is released by I-cells of the duodenum and jejunum in response to fats and amino acids.
It stimulates pancreatic acinar cells to secrete digestive enzymes and causes gallbladder contraction .
Somatostatin inhibits CCK release , thus reducing pancreatic enzyme secretion , which is the main component of exocrine pancreatic function .
✅ Therefore, somatostatin decreases pancreatic exocrine secretions primarily by inhibiting CCK .
❌ Why the Other Options Are Incorrect:
A. Pancreatic polypeptide – ❌ Incorrect
It modulates pancreatic secretions and inhibits gallbladder contraction and bile secretion.
Not a primary stimulator of exocrine enzyme release.
While somatostatin may influence it, this is not the main pathway for decreasing exocrine pancreatic secretion.
B. Secretin – ❌ Incorrect
Secreted by S-cells of the duodenum in response to acid.
Stimulates pancreatic bicarbonate secretion , not enzyme secretion.
While somatostatin may inhibit it to some extent, it’s not the main target for reducing exocrine enzyme secretion.
D. Gastrin – ❌ Incorrect
Secreted by G-cells of the stomach.
Stimulates gastric acid secretion, not pancreatic enzymes.
Inhibited by somatostatin, but not linked to pancreatic exocrine secretion .
E. Vasoactive intestinal peptide (VIP) – ❌ Incorrect
A neuropeptide that causes intestinal smooth muscle relaxation and water/electrolyte secretion.
Plays a role in intestinal secretion , not pancreatic enzyme production .
Not the primary hormone related to pancreatic exocrine enzyme secretion.
Which neurotransmitter naturally suppresses prolactin — and what drug mimics it to shrink prolactin-secreting tumors?
26 / 81
Tags:
2023
A young female presents to her gynecologist complaining of irregular menstrual periods, galactorrhea, and decreased libido. She also reports experiencing headaches and vision changes (visual field defects). Physical examination reveals no abnormalities. A blood test is performed and prolactin levels are 800 ng/mL. What is the best management for this patient?
This young woman presents with:
Menstrual irregularities
Galactorrhea (milk discharge without pregnancy)
Low libido
Headache and visual field defects
Markedly elevated prolactin level (800 ng/mL)
These are classic signs of a prolactinoma — a prolactin-secreting pituitary adenoma , likely macroadenoma given the visual symptoms.
First-line treatment for prolactinomas is a dopamine agonist , such as bromocriptine or cabergoline . Dopamine inhibits prolactin secretion from the pituitary.
Bromocriptine:
Surgery is reserved for cases unresponsive to medical therapy or with acute visual deterioration.
❌ Why the Other Options Are Incorrect
Observation alone Not appropriate — prolactin is very high and patient has symptomatic mass effect (vision changes).
Transsphenoidal pituitary resection Considered second-line if medical therapy fails or if tumor is compressing optic chiasm severely.
Glucagon Has no role in prolactinoma treatment.
Somatostatin Inhibits GH and other hormones , but is not effective for prolactinomas.
Which system is most sensitive to metabolic stimulation, such that sudden hormone replacement could overload its oxygen demand?
27 / 81
Tags:
2023
In which of the following diseases the dose of levothyroxine is reduced?
Levothyroxine is a synthetic form of thyroxine (T4) used to treat hypothyroidism . While it’s essential to restore normal thyroid hormone levels, in certain conditions, particularly ischemic heart disease , caution is required.
In ischemic heart disease (IHD) — such as angina or post-myocardial infarction — thyroid hormones can increase myocardial oxygen demand by:
Raising heart rate
Increasing contractility
Raising metabolic rate
Starting full-dose levothyroxine in such patients can precipitate angina , arrhythmias , or even heart failure . ➡️ Dose should be started low and titrated slowly under cardiac monitoring.
❌ Why the Other Options Are Incorrect
Neurological disorders These don’t generally require dose reduction unless there’s an elderly patient with comorbidities.
Respiratory disorders Thyroid hormone doesn’t directly worsen most respiratory diseases; no routine dose reduction needed.
Hematological disorders These may result from hypothyroidism (e.g., anemia), but don’t warrant reduced levothyroxine dosing.
All of these Incorrect — only ischemic heart disease clearly necessitates a lower starting dose .
When blood pressure spikes due to a surge in stress hormones, it takes more than typical meds — you need something that stops the squeeze at the receptor level.
28 / 81
Tags:
2018
Phentolamine is used to reduce hypertension in which of these?
🔹 Phentolamine is a:
Non-selective alpha-adrenergic blocker (blocks both α₁ and α₂ receptors)
Used to reduce blood pressure rapidly , especially in hypertensive crises caused by catecholamine excess
🧬 What is Pheochromocytoma?
A tumor of the adrenal medulla
Produces excessive epinephrine and norepinephrine
Causes severe paroxysmal hypertension , headaches, palpitations, sweating
🛠️ Mechanism of Phentolamine:
❌ Why the Other Options Are Incorrect:
Diabetes mellitus ❌ → Blood pressure in diabetes is managed with ACE inhibitors, ARBs , etc. → Phentolamine has no role
Hypercholesterolemia / Hypocholesterolemia ❌ → These are lipid disorders → Managed with statins, lifestyle changes , etc. → Phentolamine has no effect on cholesterol
Metabolic syndrome ❌ → Involves insulin resistance, central obesity, dyslipidemia, hypertension → Treated with lifestyle change, metformin, statins, antihypertensives , not phentolamine
Think about what determines how much hormone you need to replace — the body’s natural production, the size of the patient, and whether any systems (especially the heart) could be stressed by increasing metabolism too quickly.
29 / 81
Tags:
2018
What is the recommended initial dose of levothyroxine for patients without ischemic heart disease who require thyroid hormone replacement?
Levothyroxine is a synthetic form of thyroxine (T₄) used to treat hypothyroidism , a condition in which the thyroid gland doesn’t produce enough thyroid hormone.
When initiating therapy, the dose depends on age, weight, cardiac status , and the severity of hypothyroidism .
📌 General Dosing Strategy
Hence, 100 micrograms is typically the recommended initial dose in young, healthy, non-cardiac patients .
⚠️ Exceptions – Lower Doses Are Used In:
Elderly patients
Patients with ischemic heart disease (to avoid exacerbating angina or arrhythmias)
Patients with mild hypothyroidism or subclinical hypothyroidism
Dose adjustments are made based on TSH levels , checked every 6–8 weeks until target levels are reached.
❌ Incorrect Options Explained:
❌ 25 micrograms
❌ 50 micrograms
❌ 75 micrograms
❌ 15 micrograms
If the parasympathetic nervous system is shut down completely, what happens to secretion, digestion, urination, and heart rate — and which of these will make the skin dry?
30 / 81
Tags:
2020
Which of the following is a feature of atropine overdose?
This question tests your knowledge of anticholinergic toxicity , specifically atropine overdose — a classic medical emergency with distinct clinical features.
🔬 What is Atropine?
Atropine is a muscarinic receptor antagonist (anticholinergic).
It blocks parasympathetic nervous system activity , leading to sympathetic predominance .
📌 Classic Signs of Atropine (Anticholinergic) Toxicity:
Use the “Dry as a bone, hot as a hare, red as a beet, blind as a bat, mad as a hatter” mnemonic:
Mnemonic
Symptom
Dry as a bone
Dry skin and mucous membranes due to reduced glandular secretions
Hot as a hare
Hyperthermia (decreased sweating)
Red as a beet
Flushed skin
Blind as a bat
Mydriasis (pupil dilation) → blurred vision
Mad as a hatter
Delirium, agitation, hallucinations
Others
Urinary retention, tachycardia, constipation
➡️ So, dry skin and mucous membranes is a hallmark symptom .
❌ Why the Other Options Are Incorrect:
Option
Why Incorrect
Excessive urination
❌ Atropine causes urinary retention , not excessive urination
Sinus bradycardia
❌ Atropine increases heart rate by blocking vagal tone → tachycardia , not bradycardia
Miosis (pupil constriction)
❌ Atropine causes mydriasis (dilated pupils)
Diarrhea
❌ Anticholinergic agents like atropine decrease gut motility → constipation , not diarrhea
💊 Clinical Context:
Think about how long a long-acting hormone takes to reach a new steady state in the bloodstream. What timing would capture the first meaningful change in lab values after initiating treatment?
31 / 81
Tags:
2019
A hypothyroid patient is treated with thyroxine. How frequently should a thyroid function test be repeated?
When a patient is started on levothyroxine , monitoring thyroid function tests (TFTs) —especially TSH (thyroid-stimulating hormone) and free T4 —is crucial to:
⏱️ Why 1 Month?
Thyroxine has a long half-life (~7 days), and it takes 4–6 weeks for TSH levels to reflect the full effect of a dose adjustment.
Therefore, testing thyroid function before 4 weeks may not reflect the steady-state levels of the hormone.
Hence, 1 month (or 4–6 weeks) is the recommended time to repeat TFTs after initiating or adjusting thyroxine therapy.
❌ Explanation of Incorrect Options:
2 week interval 🔴 Too early — TSH changes won’t reflect new levels due to thyroxine’s long half-life.
6 month interval 🔴 Appropriate for stable, long-term management — not for early titration or dose adjustments.
1 year interval 🔴 Too delayed — suitable only for stable patients with no recent dose changes.
3 month interval 🔴 Acceptable for later follow-up but not soon enough after starting or adjusting therapy.
If cortisol is a matchstick, then well this glucocorticoid is a flamethrower — tiny dose, huge power. POCKET ROCKET!
32 / 81
Tags:
2024
Among the following glucocorticoids, which is considered the most potent in terms of anti-inflammatory and glucocorticoid activity ?
Glucocorticoids vary in potency depending on their anti-inflammatory strength , duration of action , and mineralocorticoid activity .
Glucocorticoid
Anti-inflammatory Potency
Sodium Retention
Duration
Notes
Cortisol
1 (baseline)
Yes
Short
Natural hormone
Corticosterone
0.3
Moderate
Short
Weak glucocorticoid
Prednisolone
~4
Minimal
Intermediate
Commonly used
Dexamethasone
~25–30 ✅
Negligible
Long
Most potent clinically
➡️ Dexamethasone is a synthetic glucocorticoid with:
That’s why it’s often used in:
❌ Why the Other Options Are Incorrect:
Cortisol ❌ Natural hormone, but much weaker (potency = 1).
Corticosterone ❌ Weak glucocorticoid, more involved in mineralocorticoid function.
Prednisolone ❌ Moderate potency (≈4× cortisol), but far less potent than dexamethasone .
Think about what the digestive system does to proteins. Would a hormone made of amino acids survive this journey intact to carry out its function?
33 / 81
Tags:
2018
Instead of taking insulin intravenously, a patient started taking insulin orally. Which of the following is most likely to happen?
🔬 Why can’t insulin be taken orally?
Insulin is a peptide hormone (a protein).
The gastrointestinal tract breaks down proteins into amino acids using:
So, if a patient takes insulin orally , it gets digested just like dietary protein and never reaches the bloodstream in active form.
🔁 Resulting Effect:
No active insulin reaches the bloodstream → glucose cannot enter cells → blood sugar rises.
Hence, persistent hyperglycemia is the expected outcome.
❌ Why the other options are incorrect:
Nausea : Not a primary or predictable effect of insulin taken orally.
Persistent hypoglycemia /Hypoglycemic shock : These require too much active insulin , but in this case, no active insulin is absorbed .
Transient ischemic attack : A neurological event unrelated to insulin intake method. No mechanism links oral insulin to this.
If you want to control blood sugar by acting before glucose even enters the bloodstream, which class of drug would you choose — one that works at the gut wall, or one that acts after glucose is already absorbed?
34 / 81
Tags:
2018
Which of the following drugs acts to inhibit glucose absorption in the small intestine?
Glucosidase inhibitors (like acarbose and miglitol ) act in the small intestine to inhibit the enzyme α-glucosidase , which is responsible for breaking down complex carbohydrates into absorbable monosaccharides like glucose.
As a result, these drugs:
Delay carbohydrate digestion
Reduce postprandial (after-meal) blood glucose spikes
Do not cause hypoglycemia when used alone
🔬 Site of Action:
Small intestinal brush border
Inhibition of α-glucosidase enzymes
Also partially inhibits pancreatic α-amylase
❌ Why the Other Options Are Incorrect:
Drug
Why Incorrect
Insulin aspart
A rapid-acting insulin analog that promotes glucose uptake , not absorption inhibition
Glipizide
A sulfonylurea that stimulates insulin secretion from pancreatic β-cells
Metformin
Decreases hepatic gluconeogenesis , improves insulin sensitivity , and slightly reduces glucose absorption , but its primary action is not in the intestine like glucosidase inhibitors
Exogenous glucose
Provides more glucose — does not inhibit absorption at all
🧪 Example of Glucosidase Inhibitors:
Acarbose
Miglitol
Taken with meals
no hormone — MIT and DIT never get married, T₃ and T₄ never born.
35 / 81
Tags:
2024
Anti-thyroid agents such as methimazole and propylthiouracil (PTU) reduce circulating levels of free T₃ and T₄ by interfering with which of the following steps in thyroid hormone synthesis?
Anti-thyroid drugs , particularly the thioamides group, act by inhibiting thyroid peroxidase (TPO) — the enzyme that catalyzes:
Oxidation of iodide to iodine
Iodination of tyrosine residues on thyroglobulin (→ MIT, DIT)
Coupling of MIT + DIT → T₃, and DIT + DIT → T₄ ✅
Thus, inhibition of the coupling reaction is the critical step by which free T₃ and T₄ levels decrease .
PTU also has an additional effect : it inhibits peripheral conversion of T₄ to T₃ (methimazole does not).
❌ Why the Other Options Are Incorrect:
Oxidation of iodide ❌ Also inhibited by thioamides, but not the main answer in the context of reducing T₃/T₄ levels directly — coupling is the key step.
Reduced uptake of iodide ❌ Seen with iodide transporter blockers (e.g., perchlorate, thiocyanate), not thioamides.
Reduced release of thyroid hormones ❌ This is the effect of high-dose iodine (Wolff-Chaikoff effect) or lithium , not thioamides.
Reduced binding to TBG ❌ TBG binding affects total hormone levels , not free T₃/T₄ synthesis. Also, antithyroid drugs do not influence TBG.
Think about whether this hormone tells the kidneys to hold on to water or let it go.
36 / 81
Tags:
2018
Which of the following statements is incorrect regarding vasopressin?
Vasopressin , also called antidiuretic hormone (ADH) , is a hormone produced by the hypothalamus and secreted by the posterior pituitary gland .
Its main action is antidiuretic — meaning it prevents the loss of water in urine . So instead of promoting diuresis (urine production) , it promotes water retention by acting on V2 receptors in the kidneys.
Hence, the statement “Promote diuresis” is incorrect — vasopressin does the opposite .
✅ Why the other options are correct:
Used in the treatment of diabetes insipidus ✅ → Especially central diabetes insipidus , where there is ADH deficiency . Vasopressin replaces it.
Used to control variceal bleeding ✅ → By causing vasoconstriction (via V1 receptors), it reduces blood flow to the portal system and controls esophageal varices bleeding.
Prevent side effects during abdominal X-ray ✅ → Sometimes used to reduce intestinal motility , limiting side effects like bowel spasms during imaging.
Help to increase blood pressure ✅ → Vasopressin causes vasoconstriction , especially in shock states, thus raising blood pressure .
When a side effect arises due to hormonal interference from a medication, think about whether a structurally or mechanistically different drug could achieve the same goal—without the unwanted hormonal effect.
37 / 81
Tags:
2018
If a patient taking spironolactone develops gynecomastia as an adverse effect, what is the recommended management?
Let’s explore this from a clinical pharmacology and patient management perspective.
🩺 Background: Spironolactone and Gynecomastia
Spironolactone is a potassium-sparing diuretic and a mineralocorticoid receptor antagonist . It is commonly used in:
It also has anti-androgenic effects because it:
Blocks androgen receptors
Inhibits testosterone synthesis
Increases peripheral conversion of testosterone to estradiol
As a result, gynecomastia (benign male breast enlargement) can occur — particularly in men on higher doses or long-term therapy.
✅ Why “Switch to Amiloride” is Correct:
Amiloride is another potassium-sparing diuretic that:
Works via blocking epithelial sodium channels (ENaCs) in the distal nephron .
Does not have anti-androgenic effects .
Maintains similar natriuretic and potassium-sparing effects as spironolactone.
➡️ So, switching to amiloride allows for continued therapeutic effect without the risk of gynecomastia .
❌ Why the Other Options Are Incorrect:
❌ Reduce the dose of spironolactone
May lessen symptoms but does not reliably reverse gynecomastia .
If the anti-androgenic effect persists even at lower doses, the problem may continue.
❌ Discontinue spironolactone and monitor for resolution
Stopping the drug may lead to loss of therapeutic benefit in managing heart failure or hyperaldosteronism.
While it might reverse gynecomastia, it’s not the most balanced approach if alternatives like amiloride are available.
❌ Perform surgery to remove excess breast tissue
❌ Start androgen replacement
When your body is growing too much due to a hormone, the best drugs are either mimics of its natural inhibitor or blockers of its receptor. Which choice offers both?
38 / 81
Tags:
2018
Which of the following drug is used to treat acromegaly?
Acromegaly is caused by excess growth hormone (GH) — usually from a GH-secreting pituitary adenoma . Treatment aims to reduce GH action or its downstream effects (IGF-1 ).
Two key drug classes used:
Octreotide
Pegvisomant
💊 These two together are standard medical therapy when surgery is not enough or not possible.
❌ Why the Other Options Are Incorrect:
Metformin ❌
Regular insulin ❌
Desmopressin ❌
None of them ❌
What hormone is typically missing in anovulatory cycles that can be given in a timed manner to safely trigger menstrual bleeding?
39 / 81
Tags:
2023
What is the treatment for oligomenorrhea?
Oligomenorrhea refers to infrequent or irregular menstrual cycles , typically defined as menstrual intervals longer than 35 days . It can result from various causes such as PCOS , thyroid disorders , hyperprolactinemia , or hypothalamic dysfunction .
When evaluating a patient, management depends on the underlying cause , but the basic treatment for regulating cycles and ensuring endometrial protection is:
➡️ Cyclical progesterone — administered for 10–14 days every 1–2 months:
Induces regular withdrawal bleeding
Prevents endometrial hyperplasia , which can occur due to unopposed estrogen in anovulatory women
❌ Why the Other Options Are Incorrect
None of these Incorrect — untreated oligomenorrhea can lead to infertility and endometrial hyperplasia .
Metformin Helpful in PCOS with insulin resistance , but not the first-line treatment for cycle regulation.
COCPs (Combined oral contraceptive pills) Often used for hormonal regulation , acne, and contraception, but cyclical progesterone is preferred when the goal is just to induce regular shedding without contraception.
Cyproterone acetate An anti-androgen , used in hirsutism and severe acne ; not first-line for simple oligomenorrhea.
Focus on timing and duration : If it starts working quickly and doesn’t last all day, it’s short-acting . If it’s steady and flat with minimal peak, it’s long-acting .
40 / 81
Tags:
2016
Which of the following is short-acting insulin?
Insulin preparations are classified based on their onset , peak , and duration of action . This helps tailor treatment for diabetes mellitus , depending on meal timing, glucose patterns, and patient needs.
⏱️ Types of Insulin (based on action):
Type
Example
Onset
Peak
Duration
Rapid-acting
Lispro, Aspart
~15 min
1–2 hrs
3–5 hrs
Short-acting
Regular insulin
~30–60 min
2–4 hrs
5–8 hrs
Intermediate-acting
NPH
1–2 hrs
6–12 hrs
18–24 hrs
Long-acting
Glargine, Detemir
~1 hr
None (flat)
~24 hrs
✅ Why “Regular insulin” Is Correct:
Regular insulin is short-acting .
It’s injected ~30 minutes before meals .
Its predictable peak and duration make it useful in hospital settings and diabetic ketoacidosis (DKA) management.
❌ Why the Other Options Are Incorrect:
Glyburide : Not insulin at all — it’s a sulfonylurea , which stimulates endogenous insulin secretion from pancreatic β-cells.
Insulin detemir : A long-acting insulin , used for basal (background) control , with a duration of ~20–24 hours .
Insulin glargine : Also long-acting , with no true peak , providing steady basal insulin for ~24 hours.
None of them : Incorrect — Regular insulin is clearly a short-acting insulin , so one of the options is correct.
When evaluating drug indications, ask: Does this condition benefit from vasoconstriction or antidiuretic action? If neither applies — especially in clot-related disorders — the drug is likely not indicated .
41 / 81
Tags:
2016
Which of the following is not an indication for vasopressin therapy?
🔹 What Is Vasopressin?
Also known as antidiuretic hormone (ADH) , vasopressin is a posterior pituitary hormone with two primary actions:
V1 receptor activation → vasoconstriction (hemostasis)
V2 receptor activation → increased water reabsorption in kidneys (antidiuretic effect)
Vasopressin and its analogs (e.g., desmopressin/DDAVP ) are used therapeutically in various conditions.
✅ Indications Where Vasopressin Is Used:
Esophageal varices: Vasopressin causes splanchnic vasoconstriction , reducing portal pressure and controlling bleeding.
Diabetes Insipidus (central type): Vasopressin (or desmopressin) replaces deficient ADH , reducing excessive urination.
Diverticular bleeding: As a vasoconstrictor , it can help manage GI bleeding including that from diverticula.
Von Willebrand disease: Desmopressin , a synthetic analog, increases release of vWF and factor VIII from endothelial stores — useful in mild cases.
❌ Why “Venous Thromboembolism” Is Incorrect:
Vasopressin is not used to treat or prevent VTE.
VTE management relies on anticoagulants (e.g., heparin, warfarin, DOACs), not vasoconstrictors .
In fact, vasopressin could theoretically worsen vascular constriction , which is not beneficial in thrombotic states.
When calcium climbs high, a certain substance calmly says — “Go back into bone, bhai.”
42 / 81
Tags:
2024
A patient is found to have elevated serum calcium levels . Which of the following agents is the most appropriate pharmacological treatment to lower calcium levels?
Calcitonin is a hormone secreted by parafollicular (C) cells of the thyroid gland. It lowers blood calcium by:
Inhibiting osteoclast activity → less bone resorption
Increasing calcium excretion in urine
Slightly reducing intestinal calcium absorption
It is used in:
❌ Why the Other Options Are Incorrect:
Vitamin D ❌ Increases calcium absorption from the gut — would worsen hypercalcemia .
Hydrochlorothiazide ❌ A thiazide diuretic that reduces calcium excretion in urine — can cause or worsen hypercalcemia .
Teriparatide ❌ A recombinant PTH analog used to increase bone formation in osteoporosis — raises serum calcium .
Think of a condition where promoting clotting and vasoconstriction would actually be dangerous , not helpful. That’s your odd one out.
43 / 81
Tags:
2018
Which of the following is not an indication for vasopressin therapy?
Vasopressin (ADH) is a hormone with vasoconstrictive and antidiuretic effects. It acts on:
📌 Common indications for vasopressin therapy:
Diabetes Insipidus (central) → Replaces deficient ADH to stop excessive urination.
Esophageal Varices → Vasoconstricts splanchnic circulation to reduce portal pressure and bleeding.
Diverticular Bleeding / GI Bleeds → Used for its vasoconstrictive effects to control bleeding.
Von Willebrand Disease (mild) → Stimulates release of von Willebrand factor and factor VIII (via desmopressin , a vasopressin analog).
❌ Why the Incorrect Option is… Correctly Incorrect:
In chronic conditions like hypothyroidism, would you prefer a drug that wears off quickly or one that provides stable hormone levels with less frequent dosing?
44 / 81
Tags:
2018
Which of the following is inappropriate about levothyroxine?
Levothyroxine (T₄) is a synthetic form of thyroxine , the primary hormone produced by the thyroid gland. It is the standard treatment for hypothyroidism and works by replacing deficient levels of circulating T₄, which gets converted into the active hormone T₃ (liothyronine) in peripheral tissues.
✅ Accurate Statements:
First-line therapy for hypothyroidism → Levothyroxine is the preferred drug due to its predictable absorption , long half-life , and stable hormone levels .
Slower acting than liothyronine → True. Levothyroxine (T₄) is a prohormone and requires conversion to T₃ , hence its onset is slower.
Longer half-life than liothyronine → Correct. T₄’s half-life is about 7 days , while T₃’s is only 1 day . This allows for once-daily dosing and more stable serum levels.
Lower doses in elderly and myxedema → True. In older patients or those with longstanding untreated hypothyroidism (e.g., myxedema), initiating at a lower dose avoids cardiac strain and adverse effects .
❌ Inaccurate Statement (Correct Answer):
Which steroid would you choose if you needed the maximum anti-inflammatory effect with minimal fluid retention ?
45 / 81
Tags:
2022
Which of the following is the most potent corticosteroid?
Corticosteroids vary in potency , duration , and mineralocorticoid activity . Among them, dexamethasone stands out as the most potent glucocorticoid , used especially when a strong anti-inflammatory or immunosuppressive effect is required.
Let’s compare a few key corticosteroids by their glucocorticoid potency :
Steroid
Relative Glucocorticoid Potency
Mineralocorticoid Activity
Cortisol (Hydrocortisone)
1
1
Cortisone
0.8
0.8
Prednisone
4
0.8
Dexamethasone
25–30
~0
Corticosterone
0.1
0.4
So, dexamethasone is about 25 times more potent than cortisol , and has negligible mineralocorticoid effect , making it ideal in conditions where fluid retention is undesirable (e.g. cerebral edema, brain tumors, etc.).
❌ Why the Other Options Are Incorrect:
Cortisol ❌ It’s the natural glucocorticoid but has the lowest potency on this list.
Prednisone ❌ Stronger than cortisol but still much weaker than dexamethasone .
Corticosterone ❌ Has minimal glucocorticoid potency , more involved in mineralocorticoid pathways.
Cortisone ❌ It is biologically inactive until converted to cortisol in the liver.
Think of the antihypertensive that improves insulin sensitivity and protects the kidneys .
46 / 81
Tags:
2018
Which of the following drugs are used to treat hypertension in metabolic syndrome?
Metabolic syndrome includes:
Central obesity
Insulin resistance
Hypertension
Dyslipidemia
Elevated fasting glucose
ACE-inhibitors (e.g., enalapril, lisinopril) are commonly used in metabolic syndrome because they:
Lower blood pressure effectively
Improve insulin sensitivity
Protect renal function , especially in diabetic patients
❌ Explanation of Incorrect Options:
Quinolones: ❌ Antibiotics—not used for blood pressure management.
Corticosteroids: ❌ Can worsen hypertension , insulin resistance, and weight gain—harmful in metabolic syndrome.
Aminoglycosides: ❌ Another class of antibiotics—can be nephrotoxic , not related to BP control.
None of these: ❌ Incorrect, because ACE inhibitors are definitely beneficial.
Consider which insulin formulation has delayed absorption due to protamine, allowing it to act longer than regular insulin but shorter than long-acting types.
47 / 81
Tags:
2020
Which of the following is an intermediate-acting insulin?
NPH insulin (Neutral Protamine Hagedorn) is a widely used intermediate-acting insulin preparation.
It contains insulin combined with protamine , which delays absorption and prolongs the duration of action to about 12-16 hours .
NPH is typically used to provide basal insulin coverage between meals.
❌ Why the Other Options Are Incorrect:
A. Insulin lispro: Rapid-acting insulin analogue; onset within 15 minutes.
B. Protamine zinc insulin: Long-acting insulin preparation.
C. Regular insulin: Short-acting insulin; onset within 30 minutes.
E. Insulin aspartate (Aspart): Rapid-acting insulin analogue.
Think about which drug would be chosen when a physician needs a powerful anti-inflammatory effect that lasts for more than a day , without needing frequent re-dosing or affecting salt balance much.
48 / 81
Tags:
2018
Which of the following is the long-acting glucocorticoid?
Glucocorticoids are steroid hormones used for their anti-inflammatory and immunosuppressive effects. They’re categorized based on how long they act in the body:
🔄 Classification by Duration of Action:
Type
Examples
Half-life & Duration
Short-acting
Hydrocortisone
~8–12 hrs
Intermediate
Prednisone, Methylprednisolone
~12–36 hrs
Long-acting
Dexamethasone
>36 hrs
🧪 Why is Dexamethasone long-acting?
It has a very high glucocorticoid potency .
Minimal mineralocorticoid (salt-retaining) effect.
Used in situations where long-lasting anti-inflammatory or brain edema-reducing effect is needed (like in cerebral edema , COVID-19 , brain tumors , etc.)
❌ Why the Other Options Are Incorrect:
Prednisone ❌ → Intermediate-acting , not long-acting. Half-life is ~12–36 hours.
Hydrocortisone ❌ → Short-acting. Often used as replacement therapy in adrenal insufficiency.
Methylprednisolone ❌ → Also intermediate-acting. Used IV in emergencies but doesn’t last as long as dexamethasone.
None of them ❌ → Incorrect. Because dexamethasone is indeed a long-acting glucocorticoid .
Which condition involves excessive urination due to failure of the kidneys to retain water—not because of blood sugar, but due to a missing hormone that acts on the collecting ducts?
49 / 81
Tags:
2020
Which of the following is an indication for vasopressin?
📌 What is Vasopressin?
Also known as antidiuretic hormone (ADH) .
Secreted by the posterior pituitary .
Acts on the kidneys to increase water reabsorption by binding to V2 receptors in the collecting ducts.
Also causes vasoconstriction via V1 receptors .
🩺 Indication: Diabetes Insipidus (DI)
Diabetes insipidus is characterized by increased urination (polyuria) and thirst (polydipsia) due to lack of ADH (central DI) or kidney resistance to ADH (nephrogenic DI) .
In central diabetes insipidus , there’s a deficiency of ADH , so giving vasopressin or its synthetic analog (desmopressin) replaces the missing hormone and reduces urine output .
❌ Why the Other Options Are Incorrect:
Cushing syndrome – ❌ Caused by excess cortisol , not treated with ADH. Vasopressin has no role here.
Diabetes mellitus – ❌ Involves glucose metabolism and insulin , not water regulation or ADH.
Adrenal carcinoma – ❌ A tumor affecting adrenal glands (e.g., cortisol, aldosterone), unrelated to ADH action.
Hypertension – ❌ While vasopressin can raise blood pressure , it’s not used therapeutically to treat hypertension; in fact, its vasoconstrictive effect would worsen it.
Sometimes the same tool that helps you see what’s wrong is also the tool that fixes the problem — especially when your target organ is hungry for what you’re offering.
50 / 81
Tags:
2018
Radioactive iodine uptake is used for which of the following?
Radioactive iodine (RAI) , typically Iodine-123 for diagnosis and Iodine-131 for treatment, is used because the thyroid gland actively takes up iodine to make thyroid hormones. This property is exploited for both:
🔬 1. Diagnosis:
RAI Uptake Scan (RAIU) is used to evaluate thyroid function and identify the cause of hyperthyroidism . The scan shows how much iodine the thyroid takes up:
High uptake → Graves’ disease, toxic multinodular goiter
Low uptake → Thyroiditis, exogenous hormone use
It also helps detect:
Hot vs cold nodules
Thyroid autonomy
Ectopic thyroid tissue
💊 2. Treatment:
Radioactive iodine therapy (usually I-131) is used to destroy thyroid tissue in:
RAI is absorbed by thyroid cells and emits beta particles , which selectively destroy overactive or malignant thyroid tissue.
❌ Why the Other Options Are Incorrect:
Diagnosis of thyroid disorders only ❌ → Incorrect — RAI is also used therapeutically.
Treatment of thyroid disorders only ❌ → Incorrect — RAI uptake is also used diagnostically.
Checking presence of drug ❌ → RAI is not used to detect drugs.
None of them ❌ → Incorrect — RAI is a core tool in both diagnosis and treatment of thyroid disease.
Consider which hormone deficiency would lead to hypotension, electrolyte imbalances, hypoglycemia, and shock-like symptoms . What hormone must be urgently replaced to stabilize the patient , especially if the problem is related to the adrenal glands ?
51 / 81
Tags:
2017
A 16-year-old girl came to the clinic with the complaints of lethargy and weight loss since six months. She also has hypotension and altered mentation. She also has mild flank tendernes. What is the treatment of choice?
Why IV Hydrocortisone is the Treatment of Choice:
The clinical picture suggests acute adrenal insufficiency (Addisonian crisis) superimposed on chronic adrenal insufficiency .
Hydrocortisone is the first-line treatment because it provides glucocorticoid replacement , which is essential for:
Maintaining blood pressure (by sensitizing vasculature to catecholamines)
Maintaining blood sugar (prevents hypoglycemia)
Reducing inflammation and stress response
In high doses , hydrocortisone also has mineralocorticoid activity , helping with sodium retention and potassium excretion .
Standard Emergency Treatment:
Immediate IV hydrocortisone
IV fluids (normal saline or dextrose) to correct hypotension and dehydration .
Glucose correction if needed (due to hypoglycemia).
Identify and treat any underlying cause (e.g., infection, stopping steroids abruptly).
Why the Other Options Are Incorrect:
Option
Why It’s Incorrect
Insulin
❌ The patient is hypoglycemic , not hyperglycemic, so insulin would worsen her condition.
Antibiotics
❌ Use only if there’s a confirmed infection . The primary issue is adrenal failure , so steroids are priority.
Sedatives
❌ May worsen altered mentation and hypotension . Sedation is dangerous in adrenal crisis.
Fluids alone
❌ Fluids help but are not sufficient by themselves . Without steroid replacement , shock will persist.
Summary:
In a 16-year-old girl with Addisonian crisis (hypotension, lethargy, hypoglycemia, and flank tenderness) , the treatment of choice is intravenous hydrocortisone to correct glucocorticoid deficiency and stabilize the patient .
PTU breaks the wedding of MIT and DIT — no couple, no T3–T4 party.
52 / 81
Tags:
2024
Propylthiouracil (PTU) reduces thyroid hormone levels through which of the following mechanisms?
Propylthiouracil (PTU) acts by:
Inhibiting thyroid peroxidase (TPO) → This blocks:
Inhibiting peripheral conversion of T4 → T3 → Unique to PTU (methimazole does not do this)
❌ Why the Other Options Are Incorrect:
Reuptake of iodine by thyroid ❌ That’s blocked by perchlorate or thiocyanate , not PTU.
Increase valence of iodine ❌ This is not a pharmacological mechanism; iodine oxidation is actually blocked by PTU.
Reducing thyroxine-binding globulin (TBG) ❌ PTU does not affect TBG. TBG controls circulating hormone levels , not production.
Reducing thyroglobulin ❌ Thyroglobulin is the storage protein for thyroid hormone, and PTU doesn’t reduce its synthesis.
What insulin regimen most closely mimics the body’s natural insulin release pattern — offering both background and meal-related coverage?
53 / 81
Tags:
2020
A 16-year-old boy has type 1 diabetes mellitus and is on mixed insulin twice daily. His HbA1c is 9.5%. Which of the following is the best next step in management?
📌 Why the HbA1c matters:
A target HbA1c in T1DM is typically <7.0–7.5% (individualized by age).
9.5% indicates chronic hyperglycemia , putting him at risk of complications: retinopathy, nephropathy, neuropathy, etc.
🔍 Problems with mixed insulin (e.g. NPH + regular or premix):
Fixed ratio of short- and intermediate-acting insulin
Rigid meal timing required
Can result in hypoglycemia between meals or postprandial hyperglycemia
Limited flexibility for teenagers whose meal/activity patterns vary
🩺 Basal-Bolus Regimen: The Gold Standard for T1DM
❌ Why the Other Options Are Incorrect:
Put him on sliding scale of insulin – ❌
Outdated and reactive — treats high sugar after it occurs , rather than preventing it
Does not improve long-term control
Add on metformin – ❌
Increase the current dosage of insulin – ❌
Might help a little, but doesn’t solve the timing mismatch of premixed insulin
Likely to worsen hypoglycemic episodes without improving postprandial spikes
Put him on split dosage of insulin twice/thrice daily – ❌
When the kidneys begin to fail, ask yourself: which medications depend on renal clearance and could become toxic even if they’ve been tolerated before?
54 / 81
Tags:
2020
A 64-year-old male who is a known case of diabetes presents to the outpatient department for a regular follow-up. His glomerular filtration rate (GFR) is 28 ml/min and he is currently on mixed insulin and metformin. In the past six months he has had no hypoglycemic event. Which of the following is the most appropriate next step in his treatment?
This case involves a 64-year-old diabetic patient with chronic kidney disease (CKD) — GFR = 28 ml/min , which is Stage 4 CKD .
The key issue here is the use of metformin in a patient with significantly reduced renal function .
🔬 Metformin and Renal Function:
Why?
Risk of lactic acidosis increases significantly in patients with impaired renal clearance of metformin
Though rare, metformin-associated lactic acidosis (MALA) is potentially fatal
➡️ Therefore, GFR of 28 mandates stopping metformin .
✅ Why Continuing Insulin Is Appropriate:
Insulin is safe in CKD , though dose adjustments may be needed due to decreased renal insulin clearance (can increase insulin sensitivity )
This patient has had no hypoglycemic episodes , so current insulin regimen appears effective and safe
❌ Why Other Options Are Incorrect:
Decrease dose of metformin and insulin – ❌
Stop metformin and start DPP-4 inhibitor – ❌
DPP-4 inhibitors may be added in some cases, but patient is well-controlled on insulin alone
No reason to add an oral agent unless needed
Some DPP-4 inhibitors (like sitagliptin) require renal dose adjustments at this GFR
Stop both insulin and metformin, start short-acting sulfonylurea – ❌
Sulfonylureas are high-risk for hypoglycemia in CKD
Especially short-acting types like glipizide still pose a risk
This patient is doing well on insulin ; no justification to change therapy
Continue same treatment – ❌
🧪 Management Summary:
Drug
GFR < 30 ml/min
Metformin
❌ Contraindicated
Insulin
✅ Safe with monitoring
DPP-4 inhibitors
✅ Some are OK with dose adjustment
Sulfonylureas
⚠️ Risk of hypoglycemia; caution
Think about the mechanism that targets nutrient absorption rather than metabolic or hormonal modulation. How might this approach differ from medications managing underlying diseases?
55 / 81
Tags:
2021
A 58-year-old woman presents to the outpatient department for a general health checkup. Her blood pressure is 140/100 mm Hg, and her pulse rate is 89 per minute. The rest of her vitals are normal. She is 163 cm tall (5’4″) and weighs 102 kg (225 lbs). Her BMI is 38.4 kg/m². She mentions that she frequently wakes up from her sleep and her husband constantly complains about her snoring. The physician advises her to lose weight for better health, but the patient complains that she has tried multiple times and follows the advice of her nutritionist religiously. She mentions she has heard of bariatric surgery and has been considering it for some time. The physician advises trying medical therapy before moving on to surgical intervention. Which of the following medications is usually prescribed to help weight loss only?
1. Understanding the clinical scenario
The patient is obese with a BMI of 38.4 kg/m².
She has symptoms suggestive of obstructive sleep apnea (waking up frequently, snoring).
She has tried diet and lifestyle modifications but struggles with weight loss.
The physician recommends medical therapy for weight loss before considering bariatric surgery.
2. Review of each medication and its role in weight management
🔹 Sitagliptin
Class: DPP-4 inhibitor (antidiabetic)
Indication: Primarily for type 2 diabetes mellitus management.
Effect on weight: Weight neutral; does not promote weight loss.
❌ Not used solely for weight loss.
🔹 Levothyroxine
Class: Thyroid hormone replacement
Indication: Hypothyroidism
Effect on weight: Can cause weight loss if hypothyroidism is corrected, but not prescribed to lose weight in euthyroid patients due to risk of hyperthyroidism.
❌ Not used solely for weight loss.
🔹 Metformin
Class: Biguanide (antidiabetic)
Indication: Type 2 diabetes mellitus
Effect on weight: May cause modest weight loss or be weight neutral.
❌ Primarily an antidiabetic, not a dedicated weight-loss drug.
🔹 Growth hormone
Class: Hormone therapy
Indication: GH deficiency, certain wasting conditions
Effect on weight: Can increase lean body mass but not primarily used for weight loss ; can cause side effects.
❌ Not used for obesity treatment.
🔹 Orlistat
Class: Lipase inhibitor
Indication: Approved for obesity management
Mechanism: Blocks absorption of dietary fats (~30%) in the intestine, leading to reduced calorie absorption.
Effect on weight: Used specifically and only for weight loss .
✅ This is the medication prescribed solely for weight loss.
Summary Table:
Medication
Primary Use
Weight Loss Effect
Used for Weight Loss Only?
Sitagliptin
Diabetes
Weight neutral
No
Levothyroxine
Hypothyroidism
May reduce weight if hypothyroid
No
Metformin
Diabetes
Modest weight loss
No
Growth hormone
GH deficiency
Increase lean mass
No
Orlistat
Obesity
Directly promotes weight loss
Yes
Consider which steps in thyroid hormone synthesis involve iodine chemistry — and what a drug would need to block to prevent new hormone production.
56 / 81
Tags:
2018
Which of the following is the effect of propylthiouracil on thyroid hormone?
Propylthiouracil (PTU) is an antithyroid medication used primarily to treat hyperthyroidism , especially Graves’ disease and in situations like thyroid storm .
🔹 Mechanism of Action:
Inhibits thyroid peroxidase enzyme :
Inhibits peripheral conversion of T₄ to T₃ :
This is unique to PTU (compared to methimazole).
Important during thyroid storm , where rapid T₃ reduction is needed.
❌ Why the Other Options Are Incorrect:
Hyper activates Na-I symporter → PTU does not affect the sodium-iodide symporter (NIS); this is responsible for iodide uptake , and PTU acts after this step.
Hypertrophy of thyroid follicles → This is a consequence of untreated hyperthyroidism or excess TSH , not a direct effect of PTU .
Inhibit T3 release from thyroglobulin → PTU does not inhibit hormone release ; it inhibits hormone synthesis .
Inactivates thyroglobulin → PTU does not act on thyroglobulin itself; rather, it interferes with the iodination process before full hormone formation on thyroglobulin.
When choosing a treatment for a patient with both metabolic and reproductive symptoms , think about a medication that can target the root cause instead of just treating the lab numbers. Consider the pathophysiology of insulin resistance and its broader effects on the endocrine system
57 / 81
Tags:
2017
A female patient came to the clinic with the complaints of amenorrhea, increased blood sugar and obesity. She is not on any medications. Which of the following drug should be advised to her?
Biguanides (Metformin) are the first-line drug for patients with insulin resistance , which is commonly seen in both Type 2 Diabetes Mellitus and Polycystic Ovary Syndrome (PCOS) .
Mechanism of Action:
Reduces hepatic glucose production (gluconeogenesis)
Increases insulin sensitivity in peripheral tissues (muscle and fat)
Helps with weight control or mild weight loss , making it ideal for obese patients
Why is Metformin Ideal Here?
Hyperglycemia: Metformin lowers blood sugar without causing hypoglycemia.
Obesity: Metformin does not cause weight gain , unlike insulin or sulfonylureas.
Amenorrhea: In cases like PCOS, Metformin can help restore menstrual cycles by improving insulin sensitivity, which reduces hyperandrogenism and helps normalize ovulation.
Thus, Metformin addresses all three of the patient’s issues in a safe and effective manner.
Incorrect Answer Explanations:
Alpha-glucosidase Inhibitors
Insulin
Sulphonylureas
Thiazolidinediones (TZDs)
Which insulin behaves like a slow, steady drip of water from a tap—no peaks, just 24-hour background coverage?
58 / 81
Tags:
2019
Which of the following insulin preparations provides 24 hour effects?
Insulin glargine is a long-acting (basal) insulin analogue that provides steady, 24-hour glucose-lowering effects with:
It’s ideal for maintaining baseline (fasting) glucose control , especially in Type 1 and advanced Type 2 diabetes .
❌ Why the Other Options Are Incorrect:
Lispro ❌ A rapid-acting insulin ⏱ Onset: 10–15 min ⏱ Peak: 1 hour ⏱ Duration: ~3–5 hours
Aspart ❌ Another rapid-acting insulin with similar profile to lispro
Glulisine ❌ Also rapid-acting — used for post-meal spikes , not 24-hour coverage
NPH insulin ❌Intermediate-acting ⏱ Onset: 1–2 hours ⏱ Peak: 4–12 hours ⏱ Duration: 12–18 hours — doesn’t last full 24 hours and has a peak , which increases hypoglycemia risk
When time is critical and blood glucose needs to be lowered immediately , which type of insulin delivery would make the most sense?
59 / 81
Tags:
2020
Which of the following explains why regular insulin is used in a hyperglycemic emergency?
In a hyperglycemic emergency like Diabetic Ketoacidosis (DKA) or Hyperosmolar Hyperglycemic State (HHS) , the patient’s blood sugar is dangerously high and needs to be lowered quickly and safely .
That’s where regular insulin comes in.
✔️ Regular insulin is the only insulin that can be given intravenously (IV) .
When given IV , it acts very quickly — this is critical in emergencies where you need to drop glucose levels fast.
It has a short half-life and predictable action , making it easy to control and adjust during continuous infusion.
❌ Why the Other Options Are Wrong (Simple):
It does not form antibody
Human insulin is not allergic
It has a delayed onset of action
None of these
If you wanted to stop a factory from producing a product, would you cut off the raw materials, block the assembly line, or destroy the warehouse? Consider what step this drug targets in hormone production.
60 / 81
Tags:
2022
Which of the following is the effect of propylthiouracil on thyroid hormone?
Propylthiouracil (PTU) is a medication used to treat hyperthyroidism , especially in conditions like Graves’ disease . It works by directly reducing the production of thyroid hormones (T3 and T4) in the thyroid gland.
PTU inhibits two key steps in thyroid hormone synthesis:
Oxidation of iodide to iodine – This is the step where iodide (I⁻) is converted to its active form (I₂), which is essential before it can be added to the tyrosine residues on thyroglobulin.
Coupling of iodotyrosine residues – This is the step where MIT (monoiodotyrosine) and DIT (diiodotyrosine) combine to form T3 and T4 .
Both of these steps are catalyzed by the enzyme thyroid peroxidase (TPO) , and PTU blocks this enzyme.
Additionally, PTU also inhibits peripheral conversion of T4 to T3 , which is important because T3 is the more active form .
❌ Why the Other Options Are Incorrect:
Inhibit T3 release from thyroglobulin : ❌ PTU doesn’t affect the release of already-formed hormones; it blocks their formation , not their secretion .
Hyper activates Na-I symporter : ❌ PTU does not increase iodine uptake; that’s the job of TSH . The symporter brings iodide into the cell, but PTU acts after that.
Inactivates thyroglobulin : ❌ PTU doesn’t destroy thyroglobulin. Thyroglobulin is the storage protein for T3 and T4 inside the thyroid follicles.
Hypertrophy of thyroid follicles : ❌ That’s a result of excess TSH stimulation , often seen in iodine deficiency or long-term PTU use due to negative feedback , but it’s not a direct effect of PTU.
When the hormone is high but its natural byproduct is missing, consider whether the hormone came from outside the body.
61 / 81
Tags:
2018
A nurse was brought to the emergency with hypoglycemia. Investigations were carried out, which showed high insulin levels with decreased C-peptide. Which of the following is mostly likely responsible for hypoglycemia?
🔬 What do the lab findings show?
These two findings together are very telling .
💡 What is C-peptide?
When the pancreas produces insulin naturally (endogenous) , it first creates proinsulin , which is then split into:
Therefore, C-peptide is released in a 1:1 ratio with endogenous insulin
So:
Source of Insulin
Insulin
C-peptide
Endogenous
High
High
Exogenous (injected insulin)
High
Low (no C-peptide in synthetic insulin)
🧪 In this case:
✅ This is the hallmark of insulin injection–induced hypoglycemia , commonly seen in:
Healthcare professionals (e.g., nurse, as in this scenario)
Factitious disorder (Munchausen syndrome)
Suicide attempts or accidental overdoses
❌ Why the Other Options Are Incorrect:
Insulinoma ❌ → A pancreatic beta-cell tumor producing insulin → Would show high insulin AND high C-peptide
Endogenous physiological insulin production ❌ → Would also result in high C-peptide , not low
Sulfonylurea drug ❌ → Stimulates endogenous insulin secretion , so both insulin and C-peptide would be high
Metformin ❌ → Does not increase insulin secretion ; rarely causes hypoglycemia, especially when used alone
What therapy replaces the very hormone that’s missing in patients showing signs of uncontrolled catabolism and high blood sugar?
62 / 81
Tags:
2023
A 30-year-old patient presents with complaints of 5 kg weight loss, fatigue, lethargy, polyuria, and polydipsia. His blood sugar level was raised. What is the best treatment for this patient?
This 30-year-old patient presents with:
These features are classic for new-onset Type 1 Diabetes Mellitus (T1DM) or possibly severe Type 2 with catabolic features . In either case, the presence of weight loss and symptoms of hyperglycemia suggests insulin deficiency , and immediate insulin therapy is required to:
Reverse the catabolic state
Prevent diabetic ketoacidosis (DKA)
Normalize blood glucose levels
Delaying insulin in such patients can lead to dangerous metabolic complications .
❌ Why the Other Options Are Incorrect
GLP-1 receptor agonists Used in Type 2 diabetes , particularly with obesity — not first-line in symptomatic, likely insulin-deficient patients.
Glyburide (sulfonylurea) Stimulates insulin secretion — ineffective if β-cell function is significantly compromised (as in Type 1).
Thiazolidinediones Improve insulin sensitivity but act too slowly and are not suitable for acute or symptomatic hyperglycemia .
Metformin First-line in stable Type 2 diabetes , but contraindicated in acute illness or significant weight loss until insulin stabilizes the patient.
If a drug stores sugar in tissues more efficiently, think about what else might get stored along with it — especially when your patient starts swelling.
63 / 81
Tags:
2018
Which class of diabetes drug causes peripheral edema?
🔹 Thiazolidinediones (TZDs)
Examples: Pioglitazone , Rosiglitazone Mechanism:
Activate PPAR-γ receptors
Improve insulin sensitivity in adipose tissue, liver, and muscle
Lower glucose levels without increasing insulin secretion
💧 Key Side Effects of TZDs:
Peripheral edema ✅ → Due to fluid retention caused by increased renal sodium reabsorption → Also related to vascular permeability effects of PPAR-γ activation
Weight gain
Exacerbation of heart failure (contraindicated in NYHA Class III/IV)
Increased risk of fractures (especially in women)
Possible bladder cancer risk (with pioglitazone)
❌ Why the Other Options Are Incorrect:
Biguanides (e.g., Metformin) ❌ → Does not cause edema → Main side effects: GI upset , lactic acidosis (in renal failure)
DPP-4 inhibitors (e.g., Sitagliptin) ❌ → Generally well tolerated → Rarely may cause nasopharyngitis , pancreatitis , or joint pain , but not edema
Meglitinides (e.g., Repaglinide) ❌ → Increase insulin secretion , side effects similar to sulfonylureas → Risk of hypoglycemia , not edema
GLP-1 analogs (e.g., Exenatide, Liraglutide) ❌ → Common side effects: Nausea, vomiting, weight loss → Rarely associated with pancreatitis , but not edema
Think about the hormone that tells your kidneys to hang onto sodium and dump potassium. Now, consider what happens if you block that hormone — which direction does potassium go?
64 / 81
Tags:
2018
Which antihypertensive drug causes hyperkalemia?
Spironolactone is a potassium-sparing diuretic and an aldosterone antagonist . It acts on the distal convoluted tubule and collecting ducts of the nephron, where it blocks the action of aldosterone .
Aldosterone promotes:
By blocking aldosterone, spironolactone reduces potassium excretion , leading to potassium retention and hence a risk of hyperkalemia .
🔬 Clinical Relevance
❌ Why the Other Options Are Incorrect
Furosemide – Incorrect
Metolazone – Incorrect
Fludrocortisone – Incorrect
Hydrochlorothiazide – Incorrect
When evaluating hormonal feedback loops, think about which substances closely mimic natural hormones without confusing the measurements used to detect them. Which synthetic steroid gives clear results without being mistaken for cortisol itself?
65 / 81
Tags:
2021
The differentiation between ACTH (adrenocorticotropic hormone)-dependent and ACTH-independent Cushing syndrome is made by administering high doses of which of the following synthetic glucocorticoids?
1. What is Cushing Syndrome?
Cushing syndrome is a condition caused by chronic exposure to excess glucocorticoids , especially cortisol . It can be due to:
ACTH-dependent causes:
ACTH-independent causes:
2. Why measure ACTH levels?
3. Role of the High-Dose Dexamethasone Suppression Test
Dexamethasone is a potent synthetic glucocorticoid that does not interfere with cortisol assays (unlike other steroids like prednisolone).
It mimics cortisol and should suppress ACTH via negative feedback .
Interpretation:
Condition
Effect of High-Dose Dexamethasone
Cushing disease (pituitary)
Partial suppression of cortisol
Ectopic ACTH
No suppression
Adrenal tumor
Already ACTH-independent; no suppression
✅ Therefore, dexamethasone is the glucocorticoid of choice for this test.
Why other options are incorrect:
Prednisolone : Cross-reacts with cortisol assays, making it unreliable for suppression tests. ❌
Corticosterone : A natural mineralocorticoid precursor, not used diagnostically. ❌
Methylprednisolone : Also cross-reacts with cortisol assays and is not standard for suppression testing. ❌
None of these : Incorrect, since dexamethasone is the correct and standard agent. ❌
Summary:
Synthetic Steroid
Use in Suppression Tests
Reason
Dexamethasone
✅ Yes
Potent, long-acting, no assay interference
Prednisolone
❌ No
Interferes with cortisol assay
Corticosterone
❌ No
Not used clinically for this purpose
Methylprednisolone
❌ No
Also interferes with cortisol assay
Which antithyroid drug is safest to use early in pregnancy due to fewer birth defects, even though it’s not the first choice outside pregnancy?
66 / 81
Tags:
2023
A 30-year-old pregnant woman, who is a known case of Grave’s disease and has been taking medications for it came for consultation regarding her treatment during pregnancy. Which of the following will be the drug of choice for this patient?
In pregnant women with Graves’ disease , antithyroid medications are required to control maternal hyperthyroidism , which can lead to miscarriage, preterm labor , and fetal hyperthyroidism if untreated.
Propylthiouracil (PTU) is the drug of choice during the first trimester of pregnancy because:
After the first trimester, therapy may be switched to methimazole due to PTU’s potential for hepatotoxicity .
❌ Why the Other Options Are Incorrect
Carbimazole Associated with congenital malformations (e.g., aplasia cutis, choanal/esophageal atresia) if used in the first trimester .
Phenytoin An anticonvulsant , not used in thyroid disorders.
Iodine Can suppress thyroid hormone temporarily but is not safe in pregnancy due to risk of fetal goiter and hypothyroidism .
Beta blocker May be used for symptomatic relief (e.g., propranolol for tremor, tachycardia), but it does not treat the underlying hormone excess .
If a drug treats hyperthyroidism by blocking thyroid hormone production in the mother, what do you expect it to do to the fetus who depends on that same hormone?
67 / 81
Tags:
2018
A 36-year-old woman who was on medication for hyperthyroidism gives birth to a baby with fetal hypothyroidism. Which of the following drugs was she taking?
Carbimazole is an antithyroid drug that inhibits thyroid hormone synthesis by blocking thyroid peroxidase , an enzyme needed for iodination of tyrosine residues in thyroglobulin. It is used to treat hyperthyroidism , including Graves’ disease.
⚠️ Why the baby developed fetal hypothyroidism:
Carbimazole crosses the placenta .
It suppresses fetal thyroid function , leading to congenital (fetal) hypothyroidism .
This is especially dangerous in early pregnancy, as thyroid hormones are critical for fetal brain development .
❌ Why the other options are incorrect:
Propranolol : A beta-blocker used symptomatically in hyperthyroidism (e.g. for tremor, palpitations), but does not affect fetal thyroid hormone synthesis significantly.
Atenolol : Another beta-blocker; not an antithyroid drug. Also less commonly used in hyperthyroidism.
Penicillin : An antibiotic; not related to thyroid function.
Warfarin : Anticoagulant; teratogenic (can cause fetal bone abnormalities and bleeding), but does not cause hypothyroidism .
Think about which insulin you’d want working quietly in the background all day, not spiking or crashing—just keeping the blood sugar steady like a good bodyguard you never notice.
68 / 81
Tags:
2019
Which of the following insulin preparations is used for long term, 24-hour activity?
Insulin glargine is a long-acting insulin designed to provide steady 24-hour basal coverage . It has:
It mimics the body’s background insulin secretion , keeping blood sugar stable between meals and overnight. It’s often used once daily in Type 1 and Type 2 diabetes for basal glycemic control .
❌ Why the Other Options Are Incorrect:
NPH insulin ❌ Intermediate-acting. Lasts 12–18 hours , not a full 24. It has a peak and often needs twice-daily dosing .
Insulin glulisine ❌ Rapid-acting insulin. Onset in 10–15 minutes , peak at 1 hour, duration ~3–5 hours. Used for mealtime coverage , not long-term.
Insulin lispro ❌ Also rapid-acting . Acts fast, but short duration —ideal for post-meal spikes , not basal coverage.
Insulin aspart ❌ Another fast-acting insulin with a quick onset and short action—not used for 24-hour control.
When a powerful hormone-like drug is used for a long time, the body often downregulates its own natural production of the same substance. Think about what happens when external support is suddenly removed and the internal system hasn’t yet recovered.
69 / 81
Tags:
2019
A patient was on prednisone treatment for a long period of time. What is most likely to happen if the drug is withdrawn?
Prednisone is a synthetic glucocorticoid , often prescribed for inflammatory and autoimmune conditions . When taken long-term , it suppresses the hypothalamic-pituitary-adrenal (HPA) axis through negative feedback .
What Happens With Long-Term Use?
CRH (from hypothalamus) and ACTH (from pituitary) are suppressed due to high external glucocorticoid levels.
The adrenal cortex becomes atrophic over time from lack of stimulation.
If prednisone is suddenly stopped , the body cannot produce enough cortisol immediately.
Why Adrenal Crisis?
Cortisol is essential for blood pressure regulation, glucose balance, and stress response .
Adrenal crisis (also called acute adrenal insufficiency ) presents with:
Hypotension
Hypoglycemia
Vomiting, weakness, confusion
Shock, and even death if untreated
This is why steroids must be tapered , not stopped abruptly, especially after long-term use.
Incorrect Answer Explanations:
Hemolytic anemia
Incorrect : This condition involves premature destruction of red blood cells .
It is not related to steroid withdrawal ; in fact, steroids may be used to treat autoimmune hemolytic anemia .
Increased risk of infection
Hyperaldosteronism
Incorrect : Aldosterone is a mineralocorticoid , not a glucocorticoid.
Prednisone primarily affects the glucocorticoid axis , not the renin-angiotensin-aldosterone system .
Withdrawal doesn’t cause hyperaldosteronism.
Adrenal hyperplasia
Incorrect : Long-term glucocorticoid therapy suppresses ACTH , leading to adrenal atrophy , not hyperplasia.
Adrenal hyperplasia is seen in congenital adrenal hyperplasia (CAH) or ACTH-secreting tumors , not steroid withdrawal.
Imagine turning off the engine of a factory for weeks because you’re importing all its products from outside. What happens if the imports stop suddenly—before the factory has time to power back up?
70 / 81
Tags:
2019
A patient was on prednisone treatment for a long period of time. What is most likely to happen if the drug is withdrawn?
🧬 Prednisone and the HPA Axis:
Prednisone is a synthetic glucocorticoid .
When given for a long period , it mimics cortisol , causing negative feedback on the hypothalamic-pituitary-adrenal (HPA) axis .
The feedback suppresses the secretion of:
Over time, this suppression leads to atrophy of the adrenal cortex , particularly the zona fasciculata , which produces cortisol .
❗ What happens upon sudden withdrawal?
The atrophied adrenal glands cannot resume cortisol production immediately .
This results in acute cortisol deficiency , known as adrenal crisis or acute adrenal insufficiency .
🔥 Adrenal Crisis Features:
Hypotension (can be life-threatening)
Hypoglycemia
Fatigue, nausea, vomiting
Electrolyte disturbances (e.g., hyponatremia, hyperkalemia)
Shock in severe cases
That’s why patients on long-term corticosteroids must taper the dose gradually —to allow the HPA axis to recover .
❌ Why the Other Options Are Incorrect:
Hemolytic anemia
Not associated with glucocorticoid withdrawal.
Some autoimmune hemolytic anemias are treated with steroids, but stopping steroids doesn’t cause hemolysis.
Increased risk of infection
Correct during steroid use , due to immunosuppression , but not the primary concern upon withdrawal .
In fact, infection risk decreases when steroids are stopped.
Hyperaldosteronism
Involves excessive aldosterone , usually from Conn syndrome or secondary causes.
Prednisone withdrawal does not increase aldosterone production.
Aldosterone is mainly regulated by the renin-angiotensin-aldosterone system , not ACTH.
Adrenal hyperplasia
Long-term steroid use leads to adrenal atrophy , not hyperplasia.
Hyperplasia is seen in congenital adrenal hyperplasia or ACTH-secreting tumors , not in steroid withdrawal.
Which drug class operates not by immediately stimulating insulin, but by turning on genes that improve how the body responds to insulin — much like rewriting the rules rather than just turning up the volume?
71 / 81
Tags:
2018
Which of the following drugs activates the PPAR-gamma nuclear receptor?
Linagliptin
Metformin
Pioglitazone
Nateglinide
Glipizide
Understanding PPAR-γ (Peroxisome Proliferator-Activated Receptor Gamma):
PPAR-γ is a nuclear receptor that regulates gene expression related to glucose metabolism , lipid uptake , and adipocyte differentiation .
It plays a major role in improving insulin sensitivity .
Drugs that activate PPAR-γ help lower blood glucose by increasing insulin sensitivity in adipose tissue, muscle, and liver .
💊 Which drug class activates PPAR-γ?
The Thiazolidinediones (TZDs) are the class of drugs that specifically activate PPAR-γ .
Examples: Pioglitazone , Rosiglitazone
Why it’s correct:
Pioglitazone is a thiazolidinedione (TZD) .
It binds to PPAR-γ , leading to:
It acts via gene transcription , so its onset is slow but long-acting.
❌ Why the Other Options Are Incorrect:
Linagliptin
Metformin
A biguanide
Works by activating AMPK (AMP-activated protein kinase)
Decreases hepatic gluconeogenesis and increases insulin sensitivity indirectly
No direct action on PPAR-γ
Nateglinide
Glipizide
A sulfonylurea
Also increases insulin secretion by stimulating β-cells
Acts on cell membranes , not nuclear receptors
What life-threatening condition can result if the body suddenly loses a hormone it has stopped producing on its own due to long-term external replacement? Think about the stress response and what hormone is essential for it.
72 / 81
Tags:
2019
A 37-year-old female who had a kidney transplant two months ago is currently on immunosuppressants and corticosteroids. She suddenly stops taking prednisone. This puts her at an increased risk of developing which of the following?
1. Prednisone is a synthetic glucocorticoid (steroid).
2. Abrupt withdrawal of corticosteroids:
The body’s natural cortisol production has been suppressed due to negative feedback on the HPA axis.
The adrenal glands become atrophied and cannot immediately resume cortisol production .
This leads to acute adrenal insufficiency , also known as an adrenal crisis .
What is Adrenal Crisis ?
Analyzing the Options:
1. Nausea
2. Opportunistic infections
3. Adrenal crisis
4. Hemolytic anemia
5. Hyperaldosteronism
Summary:
Sudden cessation of prednisone in a patient who’s been on it for weeks to months can precipitate an adrenal crisis , due to suppressed cortisol production.
✅ Correct Answer: Adrenal crisis
In conditions involving insulin resistance or glucose intolerance, ask: Does this drug help or hurt carbohydrate metabolism? That will guide you to the safest choice.
73 / 81
Tags:
2016
Which of the following drugs are used to treat hypertension in metabolic syndrome?
🔹 What is Metabolic Syndrome?
Metabolic syndrome refers to a cluster of conditions that increase the risk for cardiovascular disease and type 2 diabetes :
Abdominal obesity
Insulin resistance
Hypertension
High triglycerides
Low HDL cholesterol
Controlling blood pressure is a key part of treatment — but some antihypertensive drugs may worsen glucose tolerance or lipid profiles , so selection is critical.
✅ Why ACE Inhibitors Are Correct:
ACE inhibitors are often first-line therapy in hypertensive patients with coexisting diabetes or metabolic syndrome .
❌ Why the Other Options Are Incorrect:
Aminoglycosides: These are antibiotics (e.g., gentamicin) with no role in hypertension management. They are nephrotoxic and ototoxic .
Quinolones: Another group of antibiotics (e.g., ciprofloxacin) — unrelated to hypertension treatment.
Corticosteroids: These worsen metabolic syndrome by:
None of these: Incorrect because ACE inhibitors are used in this scenario.
Which drug used in heart disease has such a high iodine content that it can confuse the thyroid into shutting down hormone production?
74 / 81
Tags:
2020
A 62-year-old male presents to the clinic with complaints of weight gain and lethargy. He suffers from hypertension and ischemic heart disease, for which he is taking drugs. His TSH and FT4 levels are 21 mIU/L and 0.02 ng/dL, respectively. Considering drug-induced hypothyroidism, which of the following drug might be the most common cause?
📋 Case Summary:
Elderly male with weight gain and lethargy → common signs of hypothyroidism
Known case of hypertension and ischemic heart disease
TSH = 21 mIU/L (↑ high)
Free T4 = 0.02 ng/dL (↓ low) → This is primary hypothyroidism (thyroid gland failure)
Since he is taking heart-related medications , we suspect drug-induced hypothyroidism .
💊 Why Amiodarone is the Correct Answer:
Amiodarone is a class III antiarrhythmic drug used to treat atrial fibrillation , ventricular arrhythmias , etc.
It contains iodine — each 200 mg tablet has ~75 mg iodine.
This iodine overload can disrupt thyroid hormone synthesis via the Wolff–Chaikoff effect , leading to:
Since the patient has ischemic heart disease , amiodarone is likely in use — making it the most common cause in this context.
❌ Why the Other Options Are Wrong (Simple):
Phenylbutazone
Interferon
Can affect the thyroid, especially in autoimmune patients , but is mainly used in hepatitis and cancer , not heart disease.
Sulphasalazine
Interleukin blockers
Consider a scenario where hypertension arises not from typical vascular resistance or fluid overload, but from sudden surges of a powerful endogenous substance that stimulates both alpha and beta receptors. In such a case, would standard antihypertensives suffice — or would you need something more targeted?
75 / 81
Tags:
2017
Phentolamine is used to reduce hypertension in which of the following conditions?
Phentolamine is a non-selective alpha-adrenergic antagonist — it blocks both α1 and α2 receptors . This causes:
Vasodilation (via α1 blockade), reducing peripheral vascular resistance and thus lowering blood pressure.
Increased norepinephrine release (via α2 blockade), which can lead to reflex tachycardia.
Because of these effects, phentolamine is not a routine antihypertensive . Instead, it’s reserved for special hypertensive crises , particularly those involving excess catecholamines .
🧬 Enter: Pheochromocytoma
Pheochromocytoma is a catecholamine-secreting tumor of the adrenal medulla (producing epinephrine, norepinephrine, and sometimes dopamine ). This causes:
Paroxysmal or sustained hypertension
Palpitations, sweating, headache
Treating these patients with regular beta-blockers or standard antihypertensives can worsen the crisis if alpha-blockade isn’t done first. Why?
Because alpha receptors cause vasoconstriction . If you block only beta receptors (e.g., with propranolol), you’ll leave unopposed alpha activity — leading to severe vasoconstriction and hypertensive crisis .
Hence, phentolamine (or phenoxybenzamine) is used to:
✅ Correct Answer: Pheochromocytoma
❌ Why the Other Options Are Incorrect:
Hypercholesterolemia:
Diabetes mellitus:
May lead to hypertension through vascular and renal changes, but treatment is with ACE inhibitors, ARBs, or CCBs — not alpha-blockers like phentolamine.
Metabolic syndrome:
Associated with insulin resistance, obesity, and low-grade chronic hypertension. Again, not catecholamine-driven → phentolamine isn’t used.
None of these:
Think about which arthritis medications are known to cause systemic side effects like weight gain, muscle weakness, and visual changes . Could the drug mimic a hormonal syndrome if used for a long time or at high doses?
76 / 81
Tags:
2017
A 35-year-old female came to the clinic with the complaints of muscle weakness, excessive weight gain and blurry vision. She has been taking medications to relieve her arthritic pain. Which of the following medication is responsible in this case?
Steroids (glucocorticoids like prednisone) are commonly prescribed for rheumatoid arthritis and other inflammatory conditions.
Long-term steroid use can lead to iatrogenic Cushing’s syndrome , which causes:
Weight gain (central obesity, “moon face”, “buffalo hump”)
Muscle weakness (proximal myopathy due to protein catabolism)
Blurry vision (due to steroid-induced cataracts or glaucoma )
Hyperglycemia , osteoporosis , skin thinning , and immune suppression are other known side effects.
Why the Other Options Are Incorrect:
Option
Why It’s Incorrect
Leflunomide
❌ Can cause liver toxicity and teratogenicity , but not Cushingoid symptoms .
Methotrexate
❌ Causes liver toxicity , bone marrow suppression , and mouth ulcers , but not weight gain or vision problems .
Azathioprine
❌ Can lead to bone marrow suppression and infections , but not Cushing-like symptoms .
Sulfasalazine
❌ May cause GI upset, rash, or hemolysis , but not the features described .
Summary:
The patient’s muscle weakness, weight gain, and blurry vision are classic side effects of long-term steroid use , which is a common medication in arthritis management
When a drug for a metabolic disorder suddenly leads to a dangerous infection-like illness, what type of immune cell might be missing — and which drug is most infamous for causing it?
77 / 81
Tags:
2020
A patient who is taking antithyroid drugs for hyperthyroidism develops sore throat and fever. Her total leukocyte count is low. Which of the following drugs is she most likely taking?
📌 Methimazole: Key Facts
Methimazole is a thionamide , used as a first-line antithyroid drug (unless contraindicated, e.g., in the first trimester of pregnancy).
It inhibits thyroid hormone synthesis by blocking thyroid peroxidase .
⚠️ Major Adverse Effect: Agranulocytosis
Agranulocytosis = dangerously low neutrophil count
Occurs in ~0.3–0.5% of patients
Most often within the first 2–3 months of therapy, but can occur anytime
Symptoms:
Sore throat , fever , malaise
Urgent sign to stop the drug and obtain urgent CBC
➡️ This is a medical emergency , and methimazole must be discontinued immediately .
❌ Why the Other Options Are Incorrect:
Radioactive iodine – ❌
Can cause hypothyroidism long-term, but not agranulocytosis
No acute leukopenia or sore throat as a typical early side effect
Propranolol – ❌
Digoxin – ❌
Cardiac glycoside used for arrhythmias and heart failure
Not used to treat hyperthyroidism , and doesn’t cause agranulocytosis
Nadolol – ❌
Another beta-blocker , like propranolol
Used for symptomatic control in hyperthyroidism, not linked to low WBCs
🩺 Clinical Takeaway:
If a patient on methimazole or propylthiouracil (PTU) presents with sore throat and fever , always suspect agranulocytosis and order a complete blood count (CBC) immediately.
In a critical moment, timing is everything. Think about which tools work immediately versus those designed to keep things steady in the long run. Would you use a slow-release option to fix an emergency?
78 / 81
Tags:
2019
A 39-year-old man with insulin-dependent diabetes mellitus is brought to the emergency department after collapsing in a shopping mall. His blood sugar is 589 mg/dL. Which of the following preparations would have the least effect on his blood sugar level?
This patient has very high blood sugar (589 mg/dL) —a hyperglycemic emergency . In such cases, the goal is rapid correction , so short-acting insulins are preferred.
Let’s understand the types of insulin based on their onset , peak , and duration :
Type
Examples
Onset
Peak
Duration
Rapid-acting
Lispro, Aspart, Glulisine
10–30 min
30–90 min
3–5 hours
Intermediate-acting
NPH
1–2 hrs
4–12 hrs
12–18 hours
Long-acting (basal)
Glargine, Detemir
1–2 hrs
No true peak
~24 hours
Insulin glargine is a long-acting, basal insulin designed to keep blood sugar stable over 24 hours , not to rapidly bring it down. It works slowly and steadily , making it the least effective in an acute hyperglycemic episode .
❌ Why the Other Options Are Incorrect:
Insulin aspart ❌ This is a rapid-acting insulin . It works quickly and is ideal for rapid blood sugar correction , especially in emergencies.
Insulin lispro ❌ Also rapid-acting , starts in 15–30 minutes. Helps quickly reduce dangerously high glucose.
Insulin glulisine ❌ Another fast-acting insulin used in DKA or hyperglycemia crises .
NPH insulin ❌ Slower than the rapid-acting insulins, but still has a moderate peak —more effective than glargine in short-term glucose reduction.
When targeting an organ that naturally pulls something from the digestive tract, consider how the body normally absorbs that substance.
79 / 81
Tags:
2018
Which of the following is wrong about radioactive iodine?
Radioactive iodine (RAI) is primarily used in the diagnosis and treatment of thyroid diseases, because iodine is selectively taken up by thyroid tissue .
🔬 RAI Forms and Use:
Purpose
Radioisotope
Route
Notes
Diagnosis
Iodine-123 (I-123)
Oral
Used in RAI uptake scans and thyroid imaging
Treatment
Iodine-131 (I-131)
Oral
Used for Graves’, toxic nodules, and thyroid cancer
✅ Both I-123 and I-131 are given orally — usually as a capsule or liquid , not intravenously.
💊 Key Facts:
RAI is absorbed in the gut , enters the bloodstream, and is taken up by the thyroid.
For treatment , typically one oral dose is sufficient — hence the “single dose” idea.
It is used for both diagnosis and therapy , depending on the isotope and dosage.
❌ Why the Other Options Are Correct (Not Wrong):
It is given orally ✅ → Correct. Standard administration route.
Used for diagnosis ✅ → Correct. I-123 is used in thyroid scans.
It is a single dose ✅ → Correct. Therapeutic doses (e.g., for Graves’ or thyroid cancer) are often given as a one-time oral dose .
None of them ❌ → Incorrect , because “It is given intravenously” is wrong , so “None of them” can’t be the right answer.
When the body gets what it needs from the outside, does it still bother making it inside? Think about how the system reacts to an excess of a hormone mimic.
80 / 81
Tags:
2022
If a person is given steroidal anti-inflammatory drugs, what will be the drug’s effect on this person?
Steroidal anti-inflammatory drugs (like prednisone or dexamethasone) mimic cortisol , the body’s natural glucocorticoid. When you give these drugs, the body senses high glucocorticoid levels through negative feedback.
As a result, the hypothalamus and anterior pituitary reduce their secretion of:
This feedback inhibition aims to prevent overproduction of endogenous cortisol.
Because the drug is acting like cortisol , the body suppresses its own hormone production—low ACTH and low CRH .
❌ Why the Other Options Are Incorrect:
High cortisol, high CRH ❌ CRH would be low due to negative feedback from high cortisol levels.
High cortisol, low ACTH ❌ Only partially correct. Cortisol is high (from the drug), ACTH is low—but CRH would also be low, so this option is incomplete.
Low cortisol, high ACTH ❌ Steroids cause high cortisol effect, not low. Also, ACTH would be suppressed.
High CRH, high ACTH ❌ Opposite of what happens—both would be suppressed by the steroid’s cortisol-like effect.
When considering insulin types, always ask: Is this for maintenance or emergency? If it’s an acute condition , you’ll need an IV insulin with rapid action , not long-acting or intermediate preparations.
81 / 81
Tags:
2016
Which of the following is inappropriate about neutral protamine hagedorn (NPH)?
🔹 What is NPH Insulin?
NPH (Neutral Protamine Hagedorn) is an intermediate-acting insulin .
It consists of insulin complexed with protamine , which delays its absorption and prolongs its action .
Onset: 1–2 hours
Peak: 4–12 hours
Duration: 12–18 hours
It is administered subcutaneously , often twice daily .
✅ Why “It is used to treat diabetic ketoacidosis” Is Inappropriate:
NPH is not appropriate for treating diabetic ketoacidosis (DKA) .
DKA requires rapid correction of hyperglycemia , acidosis, and dehydration.
The insulin of choice in DKA is regular insulin , given IV for immediate action.
NPH has a delayed onset and variable absorption , making it unsuitable for acute emergencies like DKA.
✅ Why the Other Statements Are Correct:
It is an intermediate-acting form of insulin: True. NPH is classified as intermediate-acting.
It is given subcutaneously: True. All basal insulins, including NPH, are given subcutaneously , not IV.
None of these: Incorrect because one statement (DKA use) is wrong.
All of these: Also incorrect — only one statement is inappropriate.
Your score is
The average score is 0%
Restart quiz
Thank you for your feedback.